SlideShare uma empresa Scribd logo
1 de 70
Baixar para ler offline
MCQs FOR NURSES PREPARING
  FOR BN, B.SC.NURSING AND
  MN/M.SC. NURING ENTRANCE
            (GIFT FROM RAM SHARAN MEHTA)

                      BPKIHS, DHARAN, NEPAL

                      WISH YOU ALL THE BEST


Maternal and Child Health Nursing

1. Accompanied by her husband, a patient seeks admission to the labor and delivery area. The
    client states that she is in labor, and says she attended the hospital clinic for prenatal care.
    Which question should the nurse ask her first?
        a. “Do you have any chronic illness?”
        b. “Do you have any allergies?”
        c. “What is your expected due date?”
        d. “Who will be with you during labor?”

2. A patient is in the second stage of labor. During this stage, how frequently should the nurse in
    charge assess her uterine contractions?

       a. Every 5 minutes

       b. Every 15 minutes

       c. Every 30 minutes

       d. Every 60 minutes

3. A patient is in last trimester of pregnancy. Nurse Jane should instruct her to notify her primary
    health care provider immediately if she notices:
       a. Blurred vision
b. Hemorrhoids
       c. Increased vaginal mucus
       d. Shortness of breath on exertion

4. The nurse in charge is reviewing a patient’s prenatal history. Which finding indicates a genetic
    risk factor?

       a. The patient is 25 years old

       b. The patient has a child with cystic fibrosis

       c. The patient was exposed to rubella at 36 weeks’ gestation

       d. The patient has a history of preterm labor at 32 weeks’ gestation

5. A adult female patient is using the rhythm (calendar-basal body temperature) method of
    family planning. In this method, the unsafe period for sexual intercourse is indicated by;

       a. Return preovulatory basal body temperature

       b. Basal body temperature increase of 0.1 degrees to 0.2 degrees on the 2nd or 3rd day of
           cycle

       c. 3 full days of elevated basal body temperature and clear, thin cervical mucus

       d. Breast tenderness and mittelschmerz

6. During a nonstress test (NST), the electronic tracing displays a relatively flat line for fetal
    movement, making it difficult to evaluate the fetal heart rate (FHR). To mark the strip, the
    nurse in charge should instruct the client to push the control button at which time?

       a. At the beginning of each fetal movement

       b. At the beginning of each contraction

       c. After every three fetal movements

       d. At the end of fetal movement

7. When evaluating a client’s knowledge of symptoms to report during her pregnancy, which
    statement would indicate to the nurse in charge that the client understands the information
    given to her?

       a. “I’ll report increased frequency of urination.”

       b. “If I have blurred or double vision, I should call the clinic immediately.”
c. “If I feel tired after resting, I should report it immediately.”

       d. “Nausea should be reported immediately.”

8. When assessing a client during her first prenatal visit, the nurse discovers that the client had a
    reduction mammoplasty. The mother indicates she wants to breast-feed. What information
    should the nurse give to this mother regarding breast-feeding success?

       a. “It’s contraindicated for you to breast-feed following this type of surgery.”

       b. “I support your commitment; however, you may have to supplement each feeding with
           formula.”

       c. “You should check with your surgeon to determine whether breast-feeding would be
           possible.”

       d. “You should be able to breast-feed without difficulty.”

9. Following a precipitous delivery, examination of the client’s vagina reveals a fourth-degree
    laceration. Which of the following would be contraindicated when caring for this client?

       a. Applying cold to limit edema during the first 12 to 24 hours

       b. Instructing the client to use two or more peripads to cushion the area

       c. Instructing the client on the use of sitz baths if ordered

       d. Instructing the client about the importance of perineal (Kegel) exercises

10. A client makes a routine visit to the prenatal clinic. Although she’s 14 weeks pregnant, the
    size of her uterus approximates that in an 18- to 20-week pregnancy. Dr. Diaz diagnoses
    gestational trophoblastic disease and orders ultrasonography. The nurse expects
    ultrasonography to reveal:

       a. an empty gestational sac.

       b. grapelike clusters.

       c. a severely malformed fetus.

       d. an extrauterine pregnancy.

11. After completing a second vaginal examination of a client in labor, the nurse-midwife
    determines that the fetus is in the right occiput anterior position and at –1 station. Based on
    these findings, the nurse-midwife knows that the fetal presenting part is:
a. 1 cm below the ischial spines.

       b. directly in line with the ischial spines.

       c. 1 cm above the ischial spines.

       d. in no relationship to the ischial spines.

12. Which of the following would be inappropriate to assess in a mother who’s breast-feeding?

       a. The attachment of the baby to the breast.

       b. The mother’s comfort level with positioning the baby.

       c. Audible swallowing.

       d. The baby’s lips smacking

13. During a prenatal visit at 4 months gestation, a pregnant client asks whether tests can be done
    to identify fetal abnormalities. Between 18 and 40 weeks’ gestation, which procedure is used
    to detect fetal anomalies?

       a. Amniocentesis.

       b. Chorionic villi sampling.

       c. Fetoscopy.

       d. Ultrasound

14. A client, 30 weeks pregnant, is scheduled for a biophysical profile (BPP) to evaluate the
    health of her fetus. Her BPP score is 8. What does this score indicate?

       a. The fetus should be delivered within 24 hours.

       b. The client should repeat the test in 24 hours.

       c. The fetus isn’t in distress at this time.

       d. The client should repeat the test in 1 week.

15. A client who’s 36 weeks pregnant comes to the clinic for a prenatal checkup. To assess the
    client’s preparation for parenting, the nurse might ask which question?

       a. “Are you planning to have epidural anesthesia?”
b. “Have you begun prenatal classes?”

       c. “What changes have you made at home to get ready for the baby?”

       d. “Can you tell me about the meals you typically eat each day?”

16. A client who’s admitted to labor and delivery has the following assessment findings: gravida
    2 para 1, estimated 40 weeks’ gestation, contractions 2 minutes apart, lasting 45 seconds,
    vertex +4 station. Which of the following would be the priority at this time?

       a. Placing the client in bed to begin fetal monitoring.

       b. Preparing for immediate delivery.

       c. Checking for ruptured membranes.

       d. Providing comfort measures.

17. Nurse Roy is caring for a client in labor. The external fetal monitor shows a pattern of
    variable decelerations in fetal heart rate. What should the nurse do first?

       a. Change the client’s position.

       b. Prepare for emergency cesarean section.

       c. Check for placenta previa.

       d. Administer oxygen.

18. The nurse in charge is caring for a postpartum client who had a vaginal delivery with a
    midline episiotomy. Which nursing diagnosis takes priority for this client?

       a. Risk for deficient fluid volume related to hemorrhage

       b. Risk for infection related to the type of delivery

       c. Pain related to the type of incision

       d. Urinary retention related to periurethral edema

19. Which change would the nurse identify as a progressive physiological change in postpartum
    period?

       a. Lactation

       b. Lochia
c. Uterine involution

       d. Diuresis

20. A 39-year-old at 37 weeks’ gestation is admitted to the hospital with complaints of vaginal
    bleeding following the use of cocaine 1 hour earlier. Which complication is most likely
    causing the client’s complaint of vaginal bleeding?

       a. Placenta previa

       b. Abruptio placentae

       c. Ectopic pregnancy

       d. Spontaneous abortion

21. A client with type 1 diabetes mellitus who’s a multigravida visits the clinic at 27 weeks
    gestation. The nurse should instruct the client that for most pregnant women with type 1
    diabetes mellitus:

       a. Weekly fetal movement counts are made by the mother.

       b. Contraction stress testing is performed weekly.

       c. Induction of labor is begun at 34 weeks’ gestation.

       d. Nonstress testing is performed weekly until 32 weeks’ gestation

22. When administering magnesium sulfate to a client with preeclampsia, the nurse understands
    that this drug is given to:

       a. Prevent seizures

       b. Reduce blood pressure

       c. Slow the process of labor

       d. Increase dieresis

23. What’s the approximate time that the blastocyst spends traveling to the uterus for
    implantation?

       a. 2 days

       b. 7 days
c. 10 days

       d. 14 weeks

24. After teaching a pregnant woman who is in labor about the purpose of the episiotomy, which
 of the following purposes stated by the client would indicate to the nurse that the teaching was
 effective?

       a. Shortens the second stage of labor

       b. Enlarges the pelvic inlet

       c. Prevents perineal edema

       d. Ensures quick placenta delivery

25. A primigravida client at about 35 weeks gestation in active labor has had no prenatal care and
    admits to cocaine use during the pregnancy. Which of the following persons must the nurse
    notify?

       a. Nursing unit manager so appropriate agencies can be notified

       b. Head of the hospital’s security department

       c. Chaplain in case the fetus dies in utero

       d. Physician who will attend the delivery of the infant

26. When preparing a teaching plan for a client who is to receive a rubella vaccine during the
    postpartum period, the nurse in charge should include which of the following?

       a. The vaccine prevents a future fetus from developing congenital anomalies

       b. Pregnancy should be avoided for 3 months after the immunization

       c. The client should avoid contact with children diagnosed with rubella

       d. The injection will provide immunity against the 7-day measles.

27. A client with eclampsia begins to experience a seizure. Which of the following would the
    nurse in charge do first?

       a. Pad the side rails

       b. Place a pillow under the left buttock
c. Insert a padded tongue blade into the mouth

       d. Maintain a patent airway

28. While caring for a multigravida client in early labor in a birthing center, which of the
following foods would be best if the client requests a snack?

       a. Yogurt

       b. Cereal with milk

       c. Vegetable soup

       d. Peanut butter cookies

29. The multigravida mother with a history of rapid labor who us in active labor calls out to the
    nurse, “The baby is coming!” which of the following would be the nurse’s first action?

       a. Inspect the perineum

       b. Time the contractions

       c. Auscultate the fetal heart rate

       d. Contact the birth attendant

30. While assessing a primipara during the immediate postpartum period, the nurse in charge
    plans to use both hands to assess the client’s fundus to:

       a. Prevent uterine inversion

       b. Promote uterine involution

       c. Hasten the puerperium period

       d. Determine the size of the fundus



COMPREHENSIVES

1. Which individual is at greatest risk for developing hypertension?

       A) 45 year-old African American attorney

       B) 60 year-old Asian American shop owner
C) 40 year-old Caucasian nurse

      D) 55 year-old Hispanic teacher

2. A child who ingested 15 maximum strength acetaminophen tablets 45 minutes
ago is seen in the emergency department. Which of these orders should the nurse
do first?

      A) Gastric lavage PRN

      B) Acetylcysteine (mucomyst) for age per pharmacy

      C) Start an IV Dextrose 5% with 0.33% normal saline to keep vein open

      D) Activated charcoal per pharmacy

3. Which complication of cardiac catheterization should the nurse monitor for in
the initial 24 hours after the procedure?

      A) angina at rest

      B) thrombus formation

      C) dizziness

      D) falling blood pressure

4. A client is admitted to the emergency room with renal calculi and is complaining
of moderate to severe flank pain and nausea. The client’s temperature is 100.8
degrees Fahrenheit. The priority nursing goal for this client is

      A) Maintain fluid and electrolyte balance

      B) Control nausea

      C) Manage pain

      D) Prevent urinary tract infection

5. What would the nurse expect to see while assessing the growth of children
during their school age years?

      A) Decreasing amounts of body fat and muscle mass

      B) Little change in body appearance from year to year

      C) Progressive height increase of 4 inches each year

      D) Yearly weight gain of about 5.5 pounds per year
6. At a community health fair the blood pressure of a 62 year-old client is 160/96.
The client states “My blood pressure is usually much lower.” The nurse should tell
the client to

      A) go get a blood pressure check within the next 48 to 72 hours

      B) check blood pressure again in 2 months

      C) see the health care provider immediately

      D) visit the health care provider within 1 week for a BP check

7. The hospital has sounded the call for a disaster drill on the evening shift. Which
of these clients would the nurse put first on the list to be discharged in order to
make a room available for a new admission?

      A) A middle aged client with a history of being ventilator dependent for over 7 years
         and admitted with bacterial pneumonia five days ago

      B) A young adult with diabetes mellitus Type 2 for over 10 years and admitted with
         antibiotic induced diarrhea 24 hours ago

      C) An elderly client with a history of hypertension, hypercholesterolemia and lupus,
         and was admitted with Stevens-Johnson syndrome that morning

      D) An adolescent with a positive HIV test and admitted for acute cellulitus of the
         lower leg 48 hours ago

8. A client has been newly diagnosed with hypothyroidism and will take
levothyroxine (Synthroid) 50 mcg/day by mouth. As part of the teaching plan, the
nurse emphasizes that this medication:

      A) Should be taken in the morning

      B) May decrease the client’s energy level

      C) Must be stored in a dark container

      D) Will decrease the client’s heart rate

9. A 3 year-old child comes to the pediatric clinic after the sudden onset of
findings that include irritability, thick muffled voice, croaking on inspiration, hot to
touch, sit leaning forward, tongue protruding, drooling and suprasternal
retractions. What should the nurse do first?

      A) Prepare the child for x-ray of upper airways

      B) Examine the child’s throat

      C) Collect a sputum specimen
D) Notify the healthcare provider of the child’s status

10. In children suspected to have a diagnosis of diabetes, which one of the
following complaints would be most likely to prompt parents to take their school
age child for evaluation?

      A) Polyphagia

      B) Dehydration

      C) Bed wetting

      D) Weight loss

11. A client comes to the clinic for treatment of recurrent pelvic inflammatory
disease. The nurse recognizes that this condition most frequently follows which
type of infection?

      A) Trichomoniasis

      B) Chlamydia

      C) Staphylococcus

      D) Streptococcus

12. An RN who usually works in a spinal rehabilitation unit is floated to the
emergency department. Which of these clients should the charge nurse assign to
this RN?

      A) A middle-aged client who says "I took too many diet pills" and "my heart feels
         like it is racing out of my chest."

      B) A young adult who says "I hear songs from heaven. I need money for beer. I
         quit drinking 2 days ago for my family. Why are my arms and legs jerking?"

      C) An adolescent who has been on pain medications terminal cancer with an initial
         assessment finding pupils and a relaxed respiratory rate of 10

      D) An elderly client who reports having taken a "large crack hit" 10 minutes prior to
         walking into the emergency room

13. When teaching a client with coronary artery disease about nutrition, the nurse
should emphasize

      A) Eating 3 balanced meals a day

      B) Adding complex carbohydrates

      C) Avoiding very heavy meals
D) Limiting sodium to 7 gms per day

14. Which of these findings indicate that a pump to deliver a basal rate of 10 ml
per hour plus PRN for pain break through for morphine drip is not working?

      A) The client complains of discomfort at the IV insertion site

      B) The client states "I just can’t get relief from my pain."

      C) The level of drug is 100 ml at 8 AM and is 80 ml at noon

      D) The level of the drug is 100 ml at 8 AM and is 50 ml at noon

15. The nurse is speaking at a community meeting about personal responsibility
for health promotion. A participant asks about chiropractic treatment for illnesses.
What should be the focus of the nurse’s response?

      A) Electrical energy fields

      B) Spinal column manipulation

      C) Mind-body balance

      D) Exercise of joints

16. The nurse is performing a neurological assessment on a client post right CVA.
Which finding, if observed by the nurse, would warrant immediate attention?

      A) Decrease in level of consciousness

      B) Loss of bladder control

      C) Altered sensation to stimuli

      D) Emotional ability

17. A child who has recently been diagnosed with cystic fibrosis is in a pediatric
clinic where a nurse is performing an assessment. Which later finding of this
disease would the nurse not expect to see at this time?

      A) Positive sweat test

      B) Bulky greasy stools

      C) Moist, productive cough

      D) Meconium ileus
18. The home health nurse visits a male client to provide wound care and finds the
client lethargic and confused. His wife states he fell down the stairs 2 hours ago.
The nurse should

      A) Place a call to the client’s health care provider for instructions

      B) Send him to the emergency room for evaluation

      C) Reassure the client’s wife that the symptoms are transient

      D) Instruct the client’s wife to call the doctor if his symptoms become worse

19. Which of the following should the nurse implement to prepare a client for a
KUB (Kidney, Ureter, Bladder) radiograph test?

      A) Client must be NPO before the examination

      B) Enema to be administered prior to the examination

      C) Medicate client with Lasix 20 mg IV 30 minutes prior to the examination

      D) No special orders are necessary for this examination

20. The nurse is giving discharge teaching to a client 7 days post myocardial
infarction. He asks the nurse why he must wait 6 weeks before having sexual
intercourse. What is the best response by the nurse to this question?

      A) "You need to regain your strength before attempting such exertion."

      B) "When you can climb 2 flights of stairs without problems, it is generally safe.”

      C) "Have a glass of wine to relax you, then you can try to have sex."

      D) "If you can maintain an active walking program, you will have less risk."

21. A triage nurse has these 4 clients arrive in the emergency department within
15 minutes. Which client should the triage nurse send back to be seen first?

      A) A 2 month old infant with a history of rolling off the bed and has bulging
         fontanels with crying

      B) A teenager who got a singed beard while camping

      C) An elderly client with complaints of frequent liquid brown colored stools

      D) A middle aged client with intermittent pain behind the right scapula

22. While planning care for a toddler, the nurse teaches the parents about the
expected developmental changes for this age. Which statement by the mother
shows that she understands the child’s developmental needs?
A) "I want to protect my child from any falls."

      B) "I will set limits on exploring the house."

      C) "I understand the need to use those new skills."

      D) "I intend to keep control over our child."

23. The nurse is preparing to administer an enteral feeding to a client via a
nasogastric feeding tube. The most important action of the nurse is

      A) Verify correct placement of the tube

      B) Check that the feeding solution matches the dietary order

      C) Aspirate abdominal contents to determine the amount of last feeding remaining
         in stomach

      D) D) Ensure that feeding solution is at room temperature

24. The nurse is caring for a client with a serum potassium level of 3.5 mEq/L. The
client is placed on a cardiac monitor and receives 40 mEq KCL in 1000 ml of 5%
dextrose in water IV. Which of the following EKG patterns indicates to the nurse
that the infusions should be discontinued?

      A) Narrowed QRS complex

      B) Shortened "PR" interval

      C) Tall peaked T waves

      D) Prominent "U" waves

25. A nurse prepares to care for a 4 year-old newly admitted for
rhabdomyosarcoma. The nurse should alert the staff to pay more attention to the
function of which area of the body?

      A) All striated muscles

      B) The cerebellum

      C) The kidneys

      D) The leg bones

26. The nurse anticipates that for a family who practices Chinese medicine the
priority goal would be to

      A) Achieve harmony
B) Maintain a balance of energy

      C) Respect life

      D) Restore yin and yang

27. During an assessment of a client with cardiomyopathy, the nurse finds that the
systolic blood pressure has decreased from 145 to 110 mm Hg and the heart rate
has risen from 72 to 96 beats per minute and the client complains of periodic dizzy
spells. The nurse instructs the client to

      A) Increase fluids that are high in protein

      B) Restrict fluids

      C) Force fluids and reassess blood pressure

      D) D) Limit fluids to non-caffeine beverages

28. A client has a Swan-Ganz catheter in place. The nurse understands that this is
intended to measure

      A) Right heart function

      B) Left heart function

      C) Renal tubule function

      D) Carotid artery function

29. A nurse enters a client’s room to discover that the client has no pulse or
respirations. After calling for help, the first action the nurse should take is

      A) Start a peripheral IV

      B) Initiate closed-chest massage

      C) Establish an airway

      D) Obtain the crash cart

30. A client is receiving digoxin (Lanoxin) 0.25 mg. Daily. The health care provider
has written a new order to give metoprolol (Lopressor) 25 mg. B.I.D. In assessing
the client prior to administering the medications, which of the following should the
nurse report immediately to the health care provider?

      A) Blood pressure 94/60

      B) Heart rate 76
C) Urine output 50 ml/hour

      D) Respiratory rate 16

31. While assessing a 1 month-old infant, which finding should the nurse report
immediately?

      A) Abdominal respirations

      B) Irregular breathing rate

      C) Inspiratory grunt

      D) Increased heart rate with crying

32. The nurse practicing in a maternity setting recognizes that the post mature
fetus is at risk due to

      A) Excessive fetal weight

      B) Low blood sugar levels

      C) Depletion of subcutaneous fat

      D) Progressive placental insufficiency

33. The nurse is caring for a client who had a total hip replacement 4 days ago.
Which assessment requires the nurse’s immediate attention?

      A) I have bad muscle spasms in my lower leg of the affected extremity.

      B) "I just can’t ‘catch my breath’ over the past few minutes and I think I am in grave danger."

      C) "I have to use the bedpan to pass my water at least every 1 to 2 hours."

      D) "It seems that the pain medication is not working as well today."

34. A client has been taking furosemide (Lasix) for the past week. The nurse
recognizes which finding may indicate the client is experiencing a negative side
effect from the medication?

      A) Weight gain of 5 pounds

      B) Edema of the ankles

      C) Gastric irritability

      D) Decreased appetite
35. A client who is pregnant comes to the clinic for a first visit. The nurse gathers
data about her obstetric history, which includes 3 year-old twins at home and a
miscarriage 10 years ago at 12 weeks gestation. How would the nurse accurately
document this information?

      A) Gravida 4 para 2

      B) Gravida 2 para 1

      C) Gravida 3 para 1

      D) Gravida 3 para 2

36. The nurse is caring for a client with a venous stasis ulcer. Which nursing
intervention would be most effective in promoting healing?

      A) Apply dressing using sterile technique

      B) Improve the client’s nutrition status

      C) Initiate limb compression therapy

      D) Begin proteolytic debridement

37. A nurse is to administer meperidine hydrochloride (Demerol) 100 mg, atropine
sulfate (Atropisol) 0.4 mg, and promethizine hydrochloride (Phenergan) 50 mg IM
to a pre-operative client. Which action should the nurse take first?

      A) Raise the side rails on the bed

      B) Place the call bell within reach

      C) Instruct the client to remain in bed

      D) D) Have the client empty bladder

38. Which of these statements best describes the characteristic of an effective
reward-feedback system?

      A) Specific feedback is given as close to the event as possible

      B) Staff are given feedback in equal amounts over time

      C) Positive statements are to precede a negative statement

      D) Performance goals should be higher than what is attainable

39. A client with multiple sclerosis plans to begin an exercise program. In addition
to discussing the benefits of regular exercise, the nurse should caution the client
to avoid activities which
A) Increase the heart rate

      B) Lead to dehydration

      C) Are considered aerobic

      D) May be competitive

40. During the evaluation of the quality of home care for a client with Alzheimer’s
disease, the priority for the nurse is to reinforce which statement by a family
member?

      A) At least 2 full meals a day is eaten.

      B) We go to a group discussion every week at our community center.

      C) We have safety bars installed in the bathroom and have 24 hour alarms on the doors.

      D) The medication is not a problem to have it taken 3 times a day.




      PHARMACOLOGY

   1. The nursery nurse is putting erythromycin ointment in the newborn’s eyes to prevent
      infection. She places it in the following area of the eye:
          a. under the eyelid
          b. on the cornea.
          c. in the lower conjunctival sac
          d. by the optic disc.
   2. The physician orders penicillin for a patient with streptococcal pharyngitis. The nurse
      administers the drug as ordered, and the patient has an allergic reaction. The nurse checks
      the medication order sheet and finds that the patient is allergic to penicillin. Legal
      responsibility for the error is:
          a. only the nurse’s—she should have checked the allergies before administering the
              medication.
          b. only the physician’s—she gave the order, the nurse is obligated to follow it.
          c. only the pharmacist’s—he should alert the floor to possible allergic reactions.
          d. the pharmacist, physician, and nurse are all liable for the mistake
   3. James Perez, a nurse on a geriatric floor, is administering a dose of digoxin to one of his
      patients. The woman asks why she takes a different pill than her niece, who also has heart
      trouble. James replies that as people get older, liver and kidney function decline, and if
      the dose is as high as her niece’s, the drug will tend to:
          a. have a shorter half-life.
          b. accumulate.
          c. have decreased distribution.
          d. have increased absorption.
4. The nurse is administering Augmentin to her patient with a sinus infection. Which is the
   best way for her to insure that she is giving it to the right patient?
       a. Call the patient by name
       b. Read the name of the patient on the patient’s door
       c. Check the patient’s wristband
       d. Check the patient’s room number on the unit census list
5. The most important instructions a nurse can give a patient regarding the use of the
   antibiotic Ampicillin prescribed for her are to
       a. call the physician if she has any breathing difficulties.
       b. take it with meals so it doesn’t cause an upset stomach.
       c. take all of the medication prescribed even if the symptoms stop sooner.
       d. not share the pills with anyone else.
6. Mr. Jessie Ray, a newly admitted patient, has a seizure disorder which is being treated
   with medication. Which of the following drugs would the nurse question if ordered for
   him?
       a. Phenobarbitol, 150 mg hs
       b. Amitriptylene (Elavil), 10 mg QID.
       c. Valproic acid (Depakote), 150 mg BID
       d. Phenytoin (Dilantin), 100 mg TID
7. Mrs. Jane Gately has been dealing with uterine cancer for several months. Pain
   management is the primary focus of her current admission to your oncology unit. Her
   vital signs on admission are BP 110/64, pulse 78, respirations 18, and temperature 99.2 F.
   Morphine sulfate 6mg IV, q 4 hours, prn has been ordered. During your assessment after
   lunch, your findings are: BP 92/60, pulse 66, respirations 10, and temperature 98.8. Mrs.
   Gately is crying and tells you she is still experiencing severe pain. Your action should be
   to
       a. give her the next ordered dose of MS.
       b. give her a back rub, put on some light music, and dim the lights in the room.
       c. report your findings to the RN, requesting an alternate medication order
       d. be obtained from the physician.
       e. call her daughter to come and sit with her.
8. When counseling a patient who is starting to take MAO (monoamine oxidase) inhibitors
   such as Nardil for depression, it is essential that they be warned not to eat foods
   containing tyramine, such as:
       a. Roquefort, cheddar, or Camembert cheese.
       b. grape juice, orange juice, or raisins.
       c. onions, garlic, or scallions.
       d. ground beef, turkey, or pork.
9. The physician orders an intramuscular injection of Demerol for the postoperativepatient’s
   pain. When preparing to draw up the medication, the nurse is careful to remove the
   correct vial from the narcotics cabinet. It is labeled
       a. simethicone.
       b. albuterol.
       c. meperidine.
       d. ibuprofen.
10. The nurse is administering an antibiotic to her pediatric patient. She checks the patient’s
    armband and verifies the correct medication by checking the physician’s order,
    medication kardex, and vial. Which of the following is not considered one of the five
    “rights” of drug administration?
        a. Right dose
        b. Right route
        c. Right frequency
        d. Right time
11. A nurse is preparing the client’s morning NPH insulin dose and notices a clumpy
    precipitate inside the insulin vial. The nurse should:
        a. draw up and administer the dose
        b. shake the vial in an attempt to disperse the clumps
        c. draw the dose from a new vial
        d. warm the bottle under running water to dissolve the clump
12. A client with histoplasmosis has an order for ketoconazole (Nizoral). The nurse teaches
    the client to do which of the following while taking this medication?
        a. take the medication on an empty stomach
        b. b. take the medication with an antacid
        c. c. avoid exposure to sunlight
        d. d. limit alcohol to 2 ounces per day
13. A nurse has taught a client taking a xanthine bronchodilator about beverages to avoid.
    The nurse determines that the client understands the information if the client chooses
    which of the following beverages from the dietary menu?
        a. chocolate milk
        b. cranberry juice
        c. coffee
        d. cola
14. A client is taking famotidine (Pepcid) asks the home care nurse what would be the best
    medication to take for a headache. The nurse tells the client that it would be best to take:
        a. aspirin (acetylsalicylic acid, ASA)
        b. b. ibuprofen (Motrin)
        c. c. acetaminophen (Tylenol)
        d. d. naproxen (Naprosyn)
15. A nurse is planning dietary counseling for the client taking triamterene (Dyrenium). The
    nurse plans to include which of the following in a list of foods that are acceptable?
        a. baked potato
        b. b. bananas
        c. c. oranges
        d. d. pears canned in water
16. A client with advanced cirrhosis of the liver is not tolerating protein well, as eveidenced
    by abnormal laboratory values. The nurse anticipates that which of the following
    medications will be prescribed for the client?
        a. lactulose (Chronulac)
        b. ethacrynic acid (Edecrin)
        c. folic acid (Folvite)
        d. thiamine (Vitamin B1)
17. A female client tells the clinic nurse that her skin is very dry and irritated. Which product
    would the nurse suggest that the client apply to the dry skin?
        a. glycerin emollient
        b. aspercreame
        c. myoflex
        d. acetic acid solution
18. A nurse is providing instructions to a client regarding quinapril hydrochloride (Accupril).
    The nurse tells the client:
        a. to take the medication with food only
        b. to rise slowly from a lying to a sitting position
        c. to discontinue the medication if nausea occurs
        d. that a therapeutic effect will be noted immediately
19. Auranofin (Ridaura) is prescribed for a client with rheumatoid arthritis, and the nurse
    monitors the client for signs of an adverse effect related to the medication. Which of the
    following indicates an adverse effect?
        a. nausea
        b. b. diarrhea
        c. c. anorexia
        d. d. proteinuria
20. A client has been taking benzonatate (Tessalon) as ordered. The nurse tells the client that
    this medication should do which of the following?
        a. take away nausea and vomiting
        b. calm the persistent cough
        c. decrease anxiety level
        d. increase comfort level

           Answers:

   1. C. The ointment is placed in the lower conjunctival sac so it will not scratch the eye
   itself and will get well distributed.

   2. D. The physician, nurse, and pharmacist all are licensed professionals and share
   responsibility for errors.

  3. B. The decreased circulation to the kidney and reduced liver function tend to allow

   drugs to accumulate and have toxic effects.

   4. C. The correct way to identify a patient before giving a medication is to check the
   name on the medication administration record with the patient’s identification band. The
   nurse should also ask the patient to state their name. The name on the door or the census
   list are not sufficient proof of identification. Calling the patient by name is not as
   effective as having the patient state their name; patients may not hear well or understand
   what the nurse is saying, and may respond to a name which is not their own.
5. C. Frequently patients do not complete an entire course of antibiotic therapy, and the
bacteria are not destroyed.

   6. B. Elavil is an antidepressant that lowers the seizure threshold, so would not be
   appropriate for this patient. The other medications are anti-seizure drugs.

7. C. Morphine sulfate depresses the respiratory center. When the rate is less than 10, the MD
should be notified.

8. A. Monoamine oxidase inhibitors react with foods high in the amino acid tyramine to
cause dangerously high blood pressure. Aged cheeses are all high in this amino acid; the
other foods are not.

   9. C. The generic name for Demerol is meperidine.

   10. C. The five rights of medication administration are right drug, right dose, right route,
   right time, right patient. Frequency is not included.

   11. C. The nurse should always inspect the vial of insulin before use for solution changes
that may signify loss of potency. NPH insulin is normally uniformly cloudy. Clumping,
frosting, and precipitates are signs of insulin damage. In this situation, because potency is
questionable, it is safer to discard the vial and draw up the dose from a new vial.

   12. C. The client should be taught that ketoconazole is an antifungal medication. It should
   be taken with food or milk. Antacids should be avoided for 2 hours after it is taken
   because gastric acid is needed to activate the medication. The client should avoid
   concurrent use of alcohol, because the medication is hepatotoxic. The client should also
   avoid exposure to sunlight, because the medication increases photosensitivity.

13. B. Cola, coffee, and chocolate contain xanthine and should be avoided by the client
taking a xanthine bronchodilator. This could lead to an increased incidence of cardiovascular
and central nervous system side effects that can occur with the use of these types of
bronchodilators.

   14. C. The client is taking famotidine, a histamine receptor antagonist. This implies that
   the client has a disorder characterized by gastrointestinal (GI) irritation. The only
   medication of the ones listed in the options that is not irritating to the GI tract is
   acetaminophen. The other medications could aggravate an already existing GI problem.

15. D. Triamterene is a potassium-sparing diuretic, and clients taking this medication should
be cautioned against eating foods that are high in potassium, including many vegetables,
fruits, and fresh meats. Because potassium is very water-soluble, foods that are prepared in
water are often lower in potassium.

   16. A. The client with cirrhosis has impaired ability to metabolize protein because of liver
   dysfunction. Administration of lactulose aids in the clearance of ammonia via the
gastrointestinal (GI) tract. Ethacrynic acid is a diuretic. Folic acid and thiamine are
       vitamins, which may be used in clients with liver disease as supplemental therapy.

   17. A. Glycerin is an emollient that is used for dry, cracked, and irritated skin. Aspercreame
   and Myoflex are used to treat muscular aches. Acetic acid solution is used for irrigating,
   cleansing, and packing wounds infected by Pseudomonas aeruginosa.

       18. B. Accupril is an angiotensin-converting enzyme (ACE) inhibitor. It is used in the
       treatment of hypertension. The client should be instructed to rise slowly from a lying to
       sitting position and to permit the legs to dangle from the bed momentarily before standing
       to reduce the hypotensive effect. The medication does not need to be taken with meals. It
       may be given without regard to food. If nausea occurs, the client should be instructed to
       take a noncola carbonated beverage and salted crackers or dry toast. A full therapeutic
       effect may be noted in 1 to 2 weeks.

       19. D. Auranofin (Ridaura) is a gold preparation that is used as an antirheumatic. Gold
       toxicity is an adverse effect and is evidenced by decreased hemoglobin, leukopenia,
       reduced granulocyte counts, proteinuria, hematuria, stomatitis, glomerulonephritis,
       nephrotic syndrome, or cholestatic jaundice. Anorexia, nausea, and diarrhea are frequent
       side effects of the medication.

   20. B. Benzonatate is a locally acting antitussive. Its effectiveness is measured by the degree
   to which it decreases the intensity and frequency of cough, without eliminating the cough
   reflex.

PEDIATRIC

1. The parents of a child, age 5, who will begin school in the fall ask the nurse for anticipatory
guidance. The nurse should explain that a child of this age:
a. Still depends on the parents
b. Rebels against scheduled activities
c. Is highly sensitive to criticism
d. Loves to tattle

2. While preparing to discharge an 8-month-old infant who is recovering from gastroenteritis
and dehydration, the nurse teaches the parents about their infant’s dietary and fluid requirements.
The nurse should include which other topic in the teaching session?
a. Nursery schools
b. Toilet Training
c. Safety guidelines
d. Preparation for surgery

3. Nurse Betina should begin screening for lead poisoning when a child reaches which age?
a. 6 months
b. 12 months
c. 18 months
d. 24 months

4. When caring for an 11-month-old infant with dehydration and metabolic acidosis, the nurse
expects to see which of the following?
a. A reduced white blood cell count
b. A decreased platelet count
c. Shallow respirations
d. Tachypnea

5. After the nurse provides dietary restrictions to the parents of a child with celiac disease,
which statement by the parents indicates effective teaching?
a. “Well follow these instructions until our child’s symptoms disappear.”
b. “Our child must maintain these dietary restrictions until adulthood.”
c. “Our child must maintain these dietary restrictions lifelong.”
d. “We’ll follow these instructions until our child has completely grown and developed.”

6. A parent brings a toddler, age 19 months, to the clinic for a regular check-up. When
palpating the toddler’s fontanels, what should the nurse expects to find?
a. Closed anterior fontanel and open posterior fontanel
b. Open anterior and fontanel and closed posterior fontanel
c. Closed anterior and posterior fontanels
d. Open anterior and posterior fontanels

7. Patrick, a healthy adolescent has meningitis and is receiving I.V. and oral fluids. The nurse
should monitor this client’s fluid intake because fluid overload may cause:
a. Cerebral edema
b. Dehydration
c. Heart failure
d. Hypovolemic shock

8. An infant is hospitalized for treatment of nonorganic failure to thrive. Which nursing action
is most appropriate for this infant?
a. Encouraging the infant to hold a bottle
b. Keeping the infant on bed rest to conserve energy
c. Rotating caregivers to provide more stimulation
d. Maintaining a consistent, structured environment

9. The mother of Gian, a preschooler with spina bifida tells the nurse that her daughter sneezes
and gets a rash when playing with brightly colored balloons, and that she recently had an allergic
reaction after eating kiwifruit and bananas. The nurse would suspect that the child may have an
allergy to:
a. Bananas
b. Latex
c. Kiwifruit
d. Color dyes
10. Cristina, a mother of a 4-year-old child tells the nurse that her child is a very poor eater.
What’s the nurse’s best recommendation for helping the mother increase her child’s nutritional
intake?
a. Allow the child to feed herself
b. Use specially designed dishes for children – for example, a plate with the child’s favorite
cartoon character
c. Only serve the child’s favorite foods
d. Allow the child to eat at a small table and chair by herself

11. Nurse Roy is administering total parental nutrition (TPN) through a peripheral I.V. line to a
school-age child. What’s the smallest amount of glucose that’s considered safe and not caustic to
small veins, while also providing adequate TPN?
a. 5% glucose
b. 10% glucose
c. 15% glucose
d. 17% glucose

12. David, age 15 months, is recovering from surgery to remove Wilms’ tumor. Which
findings best indicates that the child is free from pain?
a. Decreased appetite
b. Increased heart rate
c. Decreased urine output
d. Increased interest in play

13. When planning care for a 8-year-old boy with Down syndrome, the nurse should:
a. Plan interventions according to the developmental level of a 7-year-old child because that’s
the child’s age
b. Plan interventions according to the developmental levels of a 5-year-old because the child
will have developmental delays
c. Assess the child’s current developmental level and plan care accordingly
d. Direct all teaching to the parents because the child can’t understand

14. Nurse Victoria is teaching the parents of a school-age child. Which teaching topic should
take priority?
a. Prevent accidents
b. Keeping a night light on to allay fears
c. Explaining normalcy of fears about body integrity
d. Encouraging the child to dress without help

15. The nurse is finishing her shift on the pediatric unit. Because her shift is ending, which
intervention takes top priority?
a. Changing the linens on the clients’ beds
b. Restocking the bedside supplies needed for a dressing change on the upcoming shift
c. Documenting the care provided during her shift
d. Emptying the trash cans in the assigned client room
16. Nurse Alice is providing cardiopulmonary resuscitation (CPR) to a child, age 4. the nurse
should:
a. Compress the sternum with both hands at a depth of 1½ to 2” (4 to 5 cm)
b. Deliver 12 breaths/minute
c. Perform only two-person CPR
d. Use the heel of one hand for sternal compressions

17. A 4-month-old with meningococcal meningitis has just been admitted to the pediatric unit.
Which nursing intervention has the highest priority?
a. Instituting droplet precautions
b. Administering acetaminophen (Tylenol)
c. Obtaining history information from the parents
d. Orienting the parents to the pediatric unit

18. Sheena, tells the nurse that she wants to begin toilet training her 22-month-old child. The
most important factor for the nurse to stress to the mother is:
a. Developmental readiness of the child
b. Consistency in approach
c. The mother’s positive attitude
d. Developmental level of the child’s peers

19. An infant who has been in foster care since birth requires a blood transfusion. Who is
authorized to give written, informed consent for the procedure?
a. The foster mother
b. The social worker who placed the infant in the foster home
c. The registered nurse caring for the infant
d. The nurse-manager

20. A child is undergoing remission induction therapy to treat leukemia. Allopurinol is
included in the regimen. The main reason for administering allopurinol as part of the client’s
chemotherapy regimen is to:
a. Prevent metabolic breakdown of xanthine to uric acid
b. Prevent uric acid from precipitating in the ureters
c. Enhance the production of uric acid to ensure adequate excretion of urine
d. Ensure that the chemotherapy doesn’t adversely affect the bone marrow

21. A 10-year-old client contracted severe acute respiratory syndrome (SARS) when traveling
abroad with her parents. The nurse knows she must put on personal protective equipment to
protect herself while providing care. Based on the mode of SARS transmission, which personal
protective should the nurse wear?
a. Gloves
b. Gown and gloves
c. Gown, gloves, and mask
d. Gown, gloves, mask, and eye goggles or eye shield
22. A tuberculosis intradermal skin test to detect tuberculosis infection is given to a high-risk
adolescent. How long after the test is administered should the result be evaluated?
a. Immediately
b. Within 24 hours
c. In 48 to 72 hours
d. After 5 days

23. Nurse Oliver s teaching a mother who plans to discontinue breast-feeding after 5 months.
The nurse should advise her to include which foods in her infant’s diet?
a. Iron-rich formula and baby food
b. Whole milk and baby food
c. Skim milk and baby food
d. Iron-rich formula only

24. Gracie, the mother of a 3-month-old infant calls the clinic and states that her child has a
diaper rash. What should the nurse advise?
a. “Switch to cloth diapers until the rash is gone”
b. “Use baby wipes with each diaper change.”
c. “Leave the diaper off while the infant sleeps.”
d. “Offer extra fluids to the infant until the rash improves.”

25. Nurse Kelly is teaching the parents of a young child how to handle poisoning. If the child
ingests poison, what should the parents do first?
a. Administer ipecac syrup
b. Call an ambulance immediately
c. Call the poison control center
d. Punish the child for being bad

26. A child has third-degree burns of the hands, face, and chest. Which nursing diagnosis takes
priority?
a. Ineffective airway clearance related to edema
b. Disturbed body image related to physical appearance
c. Impaired urinary elimination related to fluid loss
d. Risk for infection related to epidermal disruption

27. A 3-year-old child is receiving dextrose 5% in water and half-normal saline solution at 100
ml/hour. Which sign or symptom suggests excessive I.V. fluid intake?
a. Worsening dyspnea
b. Gastric distension
c. Nausea and vomiting
d. Temperature of 102°F (38.9° C)

28. Which finding would alert a nurse that a hospitalized 6-year-old child is at risk for a severe
asthma exacerbation?
a. Oxygen saturation of 95%
b. Mild work of breathing
c. Absence of intercostals or substernal retractions
d. History of steroid-dependent asthma

29. Nurse Mariane is caring for an infant with spina bifida. Which technique is most important
in recognizing possible hydrocephalus?
a. Measuring head circumference
b. Obtaining skull X-ray
c. Performing a lumbar puncture
d. Magnetic resonance imaging (MRI)

30. An adolescent who sustained a tibia fracture in a motor vehicle accident has a cast. What
should the nurse do to help relieve the itching?
a. Apply cool air under the cast with a blow-dryer
b. Use sterile applicators to scratch the itch
c. Apply cool water under the cast
d. Apply hydrocortisone cream under the cast using sterile applicator.

Questions:

1. Which of the following would be inappropriate when administering chemotherapy to a child?

       a. Monitoring the child for both general and specific adverse effects

       b. Observing the child for 10 minutes to note for signs of anaphylaxis

       c. Administering medication through a free-flowing intravenous line

       d. Assessing for signs of infusion infiltration and irritation

2. Which of the following is the best method for performing a physical examination on a toddler

       a. From head to toe

       b. Distally to proximally

       c. From abdomen to toes, the to head

       d. From least to most intrusive

3. Which of the following organisms is responsible for the development of rheumatic fever?

       a. Streptococcal pneumonia

       b. Haemophilus influenza

       c. Group A β-hemolytic streptococcus
d. Staphylococcus aureus

4. Which of the following is most likely associated with a cerebrovascular accident (CVA)
    resulting from congenital heart disease?

       a. Polycythemia

       b. Cardiomyopathy

       c. Endocarditis

       d. Low blood pressure

5. How does the nurse appropriately administer mycostatin suspension in an infant?

       a. Have the infant drink water, and then administer mycostatin in a syringe

       b. Place mycostatin on the nipple of the feeding bottle and have the infant suck it

       c. Mix mycostatin with formula

       d. Swab mycostatin on the affected areas

6. A mother tells the nurse that she is very worried because her 2-year old child does not finish
    his meals. What should the nurse advise the mother?

       a. make the child seat with the family in the dining room until he finishes his meal

       b. provide quiet environment for the child before meals

       c. do not give snacks to the child before meals

       d. put the child on a chair and feed him

7. The nurse is assessing a newborn who had undergone vaginal delivery. Which of the
    following findings is least likely to be observed in a normal newborn?

       a. uneven head shape

       b. respirations are irregular, abdominal, 30-60 bpm

       c. (+) moro reflex

       d. heart rate is 80 bpm

8. Which of the following situations increase risk of lead poisoning in children?
a. playing in the park with heavy traffic and with many vehicles passing by

       b. playing sand in the park

       c. playing plastic balls with other children

       d. playing with stuffed toys at home

9. An inborn error of metabolism that causes premature destruction of RBC?

       a. G6PD

       b. Hemocystinuria

       c. Phenylketonuria

       d. Celiac Disease

10. Which of the following blood study results would the nurse expect as most likely when
    caring for the child with iron deficiency anemia?

       a. Increased hemoglobin

       b. Normal hematocrit

       c. Decreased mean corpuscular volume (MCV)

       d. Normal total iron-binding capacity (TIBC)

11. The nurse answers a call bell and finds a frightened mother whose child, the patient, is
    having a seizure. Which of these actions should the nurse take?

       a. The nurse should insert a padded tongue blade in the patient’s mouth to prevent the
           child from swallowing or choking on his tongue.

       b. The nurse should help the mother restrain the child to prevent him from injuring
           himself.

       c. The nurse should call the operator to page for seizure assistance.

       d. The nurse should clear the area and position the client safely.

12. At the community center, the nurse leads an adolescent health information group, which
    often expands into other areas of discussion. She knows that these youths are trying to find
    out “who they are,” and discussion often focuses on which directions they want to take in
    school and life, as well as peer relationships. According to Erikson, this stage is known as:
a. identity vs. role confusion.

       b. adolescent rebellion.

       c. career experimentation.

       d. relationship testing

13. The nurse is assessing a 9-month-old boy for a well-baby check up. Which of the following
    observations would be of most concern?

       a. The baby cannot say “mama” when he wants his mother.

       b. The mother has not given him finger foods.

       c. The child does not sit unsupported.

       d. The baby cries whenever the mother goes out.

14. Cheska, the mother of an 11-month-old girl, KC, is in the clinic for her daughter’s
    immunizations. She expresses concern to the nurse that Shannon cannot yet walk. The nurse
    correctly replies that, according to the Denver Developmental Screen, the median age for
    walking is:

       a. 12 months.

       b. 15 months.

       c. 10 months.

       d. 14 months.

15. Sally Kent., age 13, has had a lumbar puncture to examine the CSF to determine if bacterial
    infection exists. The best position to keep her in after the procedure is:

       a. prone for two hours to prevent aspiration, should she vomit.

       b. semi-fowler’s so she can watch TV for five hours and be entertained.

       c. supine for several hours, to prevent headache.

       d. on her right sides to encourage return of CSF

16. Buck’s traction with a 10 lb. weight is securing a patient’s leg while she is waiting for
    surgery to repair a hip fracture. It is important to check circulation- sensation-movement:
a. every shift.

       b. every day.

       c. every 4 hours.

       d. every 15 minutes.

17. Carol Smith is using bronchodilators for asthma. The side effects of these drugs that you
    need to monitor this patient for include:

       a. tachycardia, nausea, vomiting, heart palpitations, inability to sleep, restlessness, and
            seizures.

       b. tachycardia, headache, dyspnea, temp . 101 F, and wheezing.

       c. blurred vision, tachycardia, hypertension, headache, insomnia, and oliguria.

       d. restlessness, insomnia, blurred vision, hypertension, chest pain, and muscle weakness.

18. The adolescent patient has symptoms of meningitis: nuchal rigidity, fever, vomiting, and
    lethargy. The nurse knows to prepare for the following test:

       a. blood culture.

       b. throat and ear culture.

       c. CAT scan.

       d. lumbar puncture.

19. The nurse is drawing blood from the diabetic patient for a glycosolated hemoglobin test. She
    explains to the woman that the test is used to determine:

       a. the highest glucose level in the past week.

       b. her insulin level.

       c. glucose levels over the past several months.

       d. her usual fasting glucose level.

20. The twelve-year-old boy has fractured his arm because of a fall from his bike. After the
    injury has been casted, the nurse knows it is most important to perform all of the following
    assessments on the area distal to the injury except:
a. capillary refill.

       b. radial and ulnar pulse.

       c. finger movement

       d. skin integrity




Answers:



1. B. When administering chemotherapy, the nurse should observe for an anaphylactic reaction
for 20 minutes and stop the medication if one is suspected. Chemotherapy is associated with both
general and specific adverse effects, therefore close monitoring for them is important.

2. D. When examining a toddler or any small child, the best way to perform the exam is from
least to most intrusive. Starting at the head or abdomen is intrusive and should be avoided.
Proceeding from distal to proximal is inappropriate at any age.

3. C. Rheumatic fever results as a delayed reaction to inadequately treated group A β-hemolytic
streptococcal infection.

4. A. The child with congenital heart disease develops polycythemia resulting from an
inadequate mechanism to compensate for decreased oxygen saturation

5. D. Mycostatin suspension is given as swab. Never mix medications with food and formula.

6. C. If the child is hungry he/she more likely would finish his meals. Therefore, the mother
should be advised not to give snacks to the child. The child is a “busy toddler.” He/she will not
able to keep still for a long time.

7. D. Normal heart rate of the newborn is 120 to 160 bpm. Choices A, B, and C are normal
assessment findings (uneven head shape is molding).

8. A. Lead poisoning may be caused by inhalation of dusk and smoke from leaded gas. It may
also be caused by lead-based paint, soil, water (especially from plumbings of old houses).

9. A. Glucose-6-phosphate dehydrogenase deficiency (G6PD) is an X-linked recessive hereditary
disease characterised by abnormally low levels of glucose-6-phosphate dehydrogenase
(abbreviated G6PD or G6PDH), a metabolic enzyme involved in the pentose phosphate pathway,
especially important in red blood cell metabolism.
10. C. For the child with iron deficiency anemia, the blood study results most likely would reveal
decreased mean corpuscular volume (MCV) which demonstrates microcytic anemia, decreased
hemoglobin, decreased hematocrit and elevated total iron binding capacity.

11. D. The primary role of the nurse when a patient has a seizure is to protect the patient from
harming him or herself.

12. A. During this period, which lasts up to the age of 18-21 years, the individual develops a
sense of “self.” Peers have a major big influence over behavior, and the major decision is to
determine a vocational goal.

13. C. Over 90% percent of babies can sit unsupported by nine months. Most babies cannot say
“mama” in the sense that it refers to their mother at this time.

14. A. By 12 months, 50 percent of children can walk well.

15. C. Lying flat keeps the patient from having a “spinal headache.” Increasing the fluid intake
will assist in replenishing the lost fluid during this time.

16. C. The patient can lose vascular status without the nurse being aware if left for more than 4
hours, yet checks should not be so frequent that the patient becomes anxious. Vital signs are
generally checked q4h, at which time the CSM checks can easily be performed.

17. A. Bronchodilators can produce the side effects listed in answer choice (A) for a short time
after the patient begins using them.

18. D. Meningitis is an infection of the meninges, the outer membrane of the brain. Since it is
surrounded by cerebrospinal fluid, a lumbar puncture will help to identify the organism involved.

19. C. The glycosolated hemoglobin test measures glucose levels for the previous 3 to 4 months.

20. D. Capillary refill, pulses, and skin temperature and color are indicative of intact circulation
and absence of compartment syndrome. Skin integrity is less important.

50 ITEM MEDICAL SURGICAL

   1. The nurse is performing her admission assessment of a patient. When grading arterial
      pulses, a 1+ pulse indicates:
          a. Above normal perfusion.
          b. Absent perfusion.
          c. Normal perfusion.
          d. Diminished perfusion.
   2. Murmurs that indicate heart disease are often accompanied by other symptoms such as:
          a. Dyspnea on exertion.
          b. Subcutaneous emphysema.
          c. Thoracic petechiae.
d. Periorbital edema.
3. Which pregnancy-related physiologic change would place the patient with a history of
    cardiac disease at the greatest risk of developing severe cardiac problems?
        a. Decrease heart rate
        b. Decreased cardiac output
        c. Increased plasma volume
        d. Increased blood pressure
4. The priority nursing diagnosis for the patient with cardiomyopathy is:
        a. Anxiety related to risk of declining health status.
        b. Ineffective individual coping related to fear of debilitating illness
        c. Fluid volume excess related to altered compensatory mechanisms.
        d. Decreased cardiac output related to reduced myocardial contractility.
5. A patient with thrombophlebitis reached her expected outcomes of care. Her affected leg
    appears pink and warm. Her pedal pulse is palpable and there is no edema present. Which
    step in the nursing process is described above?
        a. Planning
        b. Implementation
        c. Analysis
        d. Evaluation
6. An elderly patient may have sustained a basilar skull fracture after slipping and falling on
    an icy sidewalk. The nurse knows that basilar skull factures:
        a. Are the least significant type of skull fracture.
        b. May have cause cerebrospinal fluid (CSF) leaks from the nose or ears.
        c. Have no characteristic findings.
        d. Are always surgically repaired.
7. Which of the following types of drugs might be given to control increased intracranial
    pressure (ICP)?
        a. Barbiturates
        b. Carbonic anhydrase inhibitors
        c. Anticholinergics
        d. Histamine receptor blockers
8. The nurse is teaching family members of a patient with a concussion about the early signs
    of increased intracranial pressure (ICP). Which of the following would she cite as an
    early sign of increased ICP?
        a. Decreased systolic blood pressure
        b. Headache and vomiting
        c. Inability to wake the patient with noxious stimuli
        d. Dilated pupils that don’t react to light
9. Jessie James is diagnosed with retinal detachment. Which intervention is the most
    important for this patient?
        a. Admitting him to the hospital on strict bed rest
        b. Patching both of his eyes
        c. Referring him to an ophthalmologist
        d. Preparing him for surgery
10. Dr. Bruce Owen, a chemist, sustained a chemical burn to one eye. Which intervention
    takes priority for a patient with a chemical burn of the eye?
a. Patch the affected eye and call the ophthalmologist.
        b. Administer a cycloplegic agent to reduce ciliary spasm.
        c. Immediately instill a tropical anesthetic, then irrigate the eye with saline solution.
        d. Administer antibiotics to reduce the risk of infection
11. The nurse is assessing a patient and notes a Brudzinski’s sign and Kernig’s sign. These
    are two classic signs of which of the following disorders?
        a. Cerebrovascular accident (CVA)
        b. Meningitis
        c. Seizure disorder
        d. Parkinson’s disease
12. A patient is admitted to the hospital for a brain biopsy. The nurse knows that the most
    common type of primary brain tumor is:
        a. Meningioma.
        b. Angioma.
        c. Hemangioblastoma.
        d. Glioma.
13. The nurse should instruct the patient with Parkinson’s disease to avoid which of the
    following?
        a. Walking in an indoor shopping mall
        b. Sitting on the deck on a cool summer evening
        c. Walking to the car on a cold winter day
        d. Sitting on the beach in the sun on a summer day
14. Gary Jordan suffered a cerebrovascular accident that left her unable to comprehend
    speech and unable to speak. This type of aphasia is known as:
        a. Receptive aphasia
        b. Expressive aphasia
        c. Global aphasia
        d. Conduction aphasia
15. Kelly Smith complains that her headaches are occurring more frequently despite
    medications. Patients with a history of headaches should be taught to avoid:
        a. Freshly prepared meats.
        b. Citrus fruits.
        c. Skim milk
        d. Chocolate
16. Immediately following cerebral aneurysm rupture, the patient usually complains of:
        a. Photophobia
        b. Explosive headache
        c. Seizures
        d. Hemiparesis
17. Which of the following is a cause of embolic brain injury?
        a. Persistent hypertension
        b. Subarachnoid hemorrhage
        c. Atrial fibrillation
        d. Skull fracture
18. Although Ms. Priestly has a spinal cord injury, she can still have sexual intercourse.
    Discharge teaching should make her aware that:
a. She must remove indwelling urinary catheter prior to intercourse.
        b. She can no longer achieve orgasm.
        c. Positioning may be awkward.
        d. She can still get pregnant.
19. Ivy Hopkins, age 25, suffered a cervical fracture requiring immobilization with halo
    traction. When caring for the patient in halo traction, the nurse must:
        a. Keep a wrench taped to the halo vest for quick removal if cardiopulmonary
            resuscitation is necessary.
        b. Remove the brace once a day to allow the patient to rest.
        c. Encourage the patient to use a pillow under the ring.
        d. Remove the brace so that the patient can shower.
20. The nurse asks a patient’s husband if he understands why his wife is receiving
    nimodipine (Nimotop), since she suffered a cerebral aneurysm rupture. Which response
    by the husband indicates that he understands the drug’s use?
        a. “Nimodipine replaces calcium.”
        b. “Nimodipine promotes growth of blood vessels in the brain.”
        c. “Nimodipine reduces the brain’s demand for oxygen.”
        d. “Nimodipine reduces vasospasm in the brain.”
21. Many men who suffer spinal injuries continue to be sexually active. The teaching plan for
    a man with a spinal cord injury should include sexually concerns. Which of the following
    injuries would most likely prevent erection and ejaculation?
        a. C5
        b. C7
        c. T4
        d. S4
22. Cathy Bates, age 36, is a homemaker who frequently forgets to take her carbamazepine
    (Tegretol). As a result, she has been experiencing seizures. How can the nurse best help
    the patient remember to take her medication?
        a. Tell her take her medication at bedtime.
        b. Instruct her to take her medication after one of her favorite television shows.
        c. Explain that she should take her medication with breakfast.
        d. Tell her to buy an alarm watch to remind her.
23. Richard Barnes was diagnosed with pneumococcal meningitis. What response by the
    patient indicates that he understands the precautions necessary with this diagnosis?
        a. “I’m so depressed because I can’t have any visitors for a week.”
        b. “Thank goodness, I’ll only be in isolation for 24 hours.”
        c. “The nurse told me that my urine and stool are also sources of meningitis
            bacteria.”
        d. “The doctor is a good friend of mine and won’t keep me in isolation.”
24. An early symptom associated with amyotrophic lateral sclerosis (ALS) includes:
        a. Fatigue while talking
        b. Change in mental status
        c. Numbness of the hands and feet
        d. Spontaneous fractures
25. When caring for a patient with esophageal varices, the nurse knows that bleeding in this
    disorder usually stems from:
a. Esophageal perforation
         b. Pulmonary hypertension
         c. Portal hypertension
         d. Peptic ulcers
26. Tiffany Black is diagnosed with type A hepatitis. What special precautions should the
    nurse take when caring for this patient?
         a. Put on a mask and gown before entering the patient’s room.
         b. Wear gloves and a gown when removing the patient’s bedpan.
         c. Prevent the droplet spread of the organism.
         d. Use caution when bringing food to the patient.
27. Discharge instructions for a patient who has been operated on for colorectal cancer
    include irrigating the colostomy. The nurse knows her teaching is effective when the
    patient states he’ll contact the doctor if:
         a. He experiences abdominal cramping while the irrigant is infusing
         b. He has difficulty inserting the irrigation tube into the stoma
         c. He expels flatus while the return is running out
         d. He’s unable to complete the procedure in 1 hour
28. The nurse explains to the patient who has an abdominal perineal resection that an
    indwelling urinary catheter must be kept in place for several days afterward because:
         a. It prevents urinary tract infection following surgery
         b. It prevents urine retention and resulting pressure on the perineal wound
         c. It minimizes the risk of wound contamination by the urine
         d. It determines whether the surgery caused bladder trauma
29. The first day after, surgery the nurse finds no measurable fecal drainage from a patient’s
    colostomy stoma. What is the most appropriate nursing intervention?
         a. Call the doctor immediately.
         b. Obtain an order to irrigate the stoma.
         c. Place the patient on bed rest and call the doctor.
         d. Continue the current plan of care.
30. If a patient’s GI tract is functioning but he’s unable to take foods by mouth, the preferred
    method of feeding is:
         a. Total parenteral nutrition
         b. Peripheral parenteral nutrition
         c. Enteral nutrition
         d. Oral liquid supplements
31. Which type of solution causes water to shift from the cells into the plasma?
         a. Hypertonic
         b. Hypotonic
         c. Isotonic
         d. Alkaline
32. Particles move from an area of greater osmelarity to one of lesser osmolarity through:
         a. Active transport
         b. Osmosis
         c. Diffusion
         d. Filtration
33. Which assessment finding indicates dehydration?
a. Tenting of chest skin when pinched
        b. Rapid filling of hand veins
        c. A pulse that isn’t easily obliterated
        d. Neck vein distention
34. Which nursing intervention would most likely lead to a hypo-osmolar state?
        a. Performing nasogastric tube irrigation with normal saline solution
        b. Weighing the patient daily
        c. Administering tap water enema until the return is clear
        d. Encouraging the patient with excessive perspiration to dink broth
35. Which assessment finding would indicate an extracellular fluid volume deficit?
        a. Bradycardia
        b. A central venous pressure of 6 mm Hg
        c. Pitting edema
        d. An orthostatic blood pressure change
36. A patient with metabolic acidosis has a preexisting problem with the kidneys. Which
    other organ helps regulate blood pH?
        a. Liver
        b. Pancreas
        c. Lungs
        d. heart
37. The nurse considers the patient anuric if the patient;
        a. Voids during the nighttime hours
        b. Has a urine output of less than 100 ml in 24 hours
        c. Has a urine output of at least 100 ml in 2 hours
        d. Has pain and burning on urination
38. Which nursing action is appropriate to prevent infection when obtaining a sterile urine
    specimen from an indwelling urinary catheter?
        a. Aspirate urine from the tubing port using a sterile syringe and needle
        b. Disconnect the catheter from the tubing and obtain urine
        c. Open the drainage bag and pour out some urine
        d. Wear sterile gloves when obtaining urine
39. After undergoing a transurethral resection of the prostate to treat benign prostatic
    hypertrophy, a patient is retuned to the room with continuous bladder irrigation in place.
    One day later, the patient reports bladder pain. What should the nurse do first?
        a. Increase the I.V. flow rate
        b. Notify the doctor immediately
        c. Assess the irrigation catheter for patency and drainage
        d. Administer meperidine (Demerol) as prescribed
40. A patient comes to the hospital complaining of sudden onset of sharp, severe pain
    originating in the lumbar region and radiating around the side and toward the bladder.
    The patient also reports nausea and vomiting and appears pale, diaphoretic, and anxious.
    The doctor tentatively diagnoses renal calculi and orders flat-plate abdominal X-rays.
    Renal calculi can form anywhere in the urinary tract. What is their most common
    formation site?
        a. Kidney
        b. Ureter
c. Bladder
        d. Urethra
41. A patient comes to the hospital complaining of severe pain in the right flank, nausea, and
    vomiting. The doctor tentatively diagnoses right ureter-olithiasis (renal calculi). When
    planning this patient’s care, the nurse should assign highest priority to which nursing
    diagnosis?
        a. Pain
        b. Risk of infection
        c. Altered urinary elimination
        d. Altered nutrition: less than body requirements
42. The nurse is reviewing the report of a patient’s routine urinalysis. Which of the following
    values should the nurse consider abnormal?
        a. Specific gravity of 1.002
        b. Urine pH of 3
        c. Absence of protein
        d. Absence of glucose
43. A patient with suspected renal insufficiency is scheduled for a comprehensive diagnostic
    work-up. After the nurse explains the diagnostic tests, the patient asks which part of the
    kidney “does the work.” Which answer is correct?
        a. The glomerulus
        b. Bowman’s capsule
        c. The nephron
        d. The tubular system
44. During a shock state, the renin-angiotensin-aldosterone system exerts which of the
    following effects on renal function?
        a. Decreased urine output, increased reabsorption of sodium and water
        b. Decreased urine output, decreased reabsorption of sodium and water
        c. Increased urine output, increased reabsorption of sodium and water
        d. Increased urine output, decreased reabsorption of sodium and water
45. While assessing a patient who complained of lower abdominal pressure, the nurse notes a
    firm mass extending above the symphysis pubis. The nurse suspects:
        a. A urinary tract infection
        b. Renal calculi
        c. An enlarged kidney
        d. A distended bladder
46. Gregg Lohan, age 75, is admitted to the medical-surgical floor with weakness and left-
    sided chest pain. The symptoms have been present for several weeks after a viral illness.
    Which assessment finding is most symptomatic of pericarditis?
        a. Pericardial friction rub
        b. Bilateral crackles auscultated at the lung bases
        c. Pain unrelieved by a change in position
        d. Third heart sound (S3)
47. James King is admitted to the hospital with right-side-heart failure. When assessing him
    for jugular vein distention, the nurse should position him:
        a. Lying on his side with the head of the bed flat.
        b. Sitting upright.
c. Flat on his back.
        d. Lying on his back with the head of the bed elevated 30 to 45 degrees.
48. The nurse is interviewing a slightly overweight 43-year-old man with mild emphysema
    and borderline hypertension. He admits to smoking a pack of cigarettes per day. When
    developing a teaching plan, which of the following should receive highest priority to help
    decrease respiratory complications?
        a. Weight reduction
        b. Decreasing salt intake
        c. Smoking cessation
        d. Decreasing caffeine intake
49. What is the ratio of chest compressions to ventilations when one rescuer performs
    cardiopulmonary resuscitation (CPR) on an adult?
        a. 15:1
        b. 15:2
        c. 12:1
        d. 12:2
50. When assessing a patient for fluid and electrolyte balance, the nurse is aware that the
    organs most important in maintaining this balance are the:
        a. Pituitary gland and pancreas
        b. Liver and gallbladder.
        c. Brain stem and heart.
        d. Lungs and kidneys.




           MEDICAL SURGICAL ANSWERS:

1. Answer: D
   A 1+ pulse indicates weak pulses and is associated with diminished perfusion. A 4+ is
   bounding perfusion, a 3+ is increased perfusion, a 2+ is normal perfusion, and 0 is absent
   perfusion.
2. Answer: A
   A murmur that indicates heart disease is often accompanied by dyspnea on exertion,
   which is a hallmark of heart failure. Other indicators are tachycardia, syncope, and chest
   pain. Subcutaneous emphysema, thoracic petechiae, and perior-bital edema aren’t
   associated with murmurs and heart disease.
3. Answer: C
   Pregnancy increase plasma volume and expands the uterine vascular bed, possibly
   increasing both the heart rate and cardiac output. These changes may cause cardiac stress,
   especially during the second trimester. Blood pressure during early pregnancy may
   decrease, but it gradually returns to prepregnancy levels.
4. Answer: D
   Decreased cardiac output related to reduced myocardial contractility is the greatest threat
   to the survival of a patient with cardiomyopathy. The other options can be addressed once
   cardiac output and myocardial contractility have been restored.
5. Answer: D
    Evaluation assesses the effectiveness of the treatment plan by determining if the patient
    has met the expected treatment outcome. Planning refers to designing a plan of action
    that will help the nurse deliver quality patient care. Implementation refers to all of the
    nursing interventions directed toward solving the patient’s nursing problems. Analysis is
    the process of identifying the patient’s nursing problems.
6. Answer: B
    A basilar skull fracture carries the risk of complications of dural tear, causing CSF
    leakage and damage to cranial nerves I, II, VII, and VIII. Classic findings in this type of
    fracture may include otorrhea, rhinorrhea, Battle’s signs, and raccoon eyes. Surgical
    treatment isn’t always required.
7. Answer: A
    Barbiturates may be used to induce a coma in a patient with increased ICP. This
    decreases cortical activity and cerebral metabolism, reduces cerebral blood volume,
    decreases cerebral edema, and reduces the brain’s need for glucose and oxygen. Carbonic
    anhydrase inhibitors are used to decrease ocular pressure or to decrease the serum pH in a
    patient with metabolic alkalosis. Anticholinergics have many uses including reducing GI
    spasms. Histamine receptor blockers are used to decrease stomach acidity.
8. Answer: B
    Headache and projectile vomiting are early signs of increased ICP. Decreased systolic
    blood pressure, unconsciousness, and dilated pupils that don’t reac to light are considered
    late signs.
9. Answer: A
    Immediate bed rest is necessary to prevent further injury. Both eyes should be patched to
    avoid consensual eye movement and the patient should receive early referral to an
    ophthalmologist should treat the condition immediately. Retinal reattachment can be
    accomplished by surgery only. If the macula is detached or threatened, surgery is urgent;
    prolonged detachment of the macula results in permanent loss of central vision.
10. Answer: C
    A chemical burn to the eye requires immediate instillation of a topical anesthetic
    followed by irrigation with copious amounts of saline solution. Irrigation should be done
    for 5 to 10 minutes, and then the pH of the eye should be checked. Irrigation should be
    continued until the pH of the eye is restored to neutral (pH 7.0): Double eversion of the
    eyelids should be performed to look for and remove ciliary spasm, and an antibiotic
    ointment can be administered to reduce the risk of infection. Then the eye should be
    patched. Parenteral narcotic analgesia is often required for pain relief. An
    ophthalmologist should also be consulted.
11. Answer: B
    A positive response to one or both tests indicates meningeal irritation that is present with
    meningitis. Brudzinski’s and Kernig’s signs don’t occur in CVA, seizure disorder, or
    Parkinson’s disease.
12. Answer: D
    Gliomas account for approximately 45% of all brain tumors. Meningiomas are the second
    most common, with 15%. Angiomas and hemangioblastomas are types of cerebral
    vascular tumors that account for 3% of brain tumors.
13. Answer: D
    The patient with Parkinson’s disease may be hypersensitive to heat, which increases the
    risk of hyperthermia, and he should be instructed to avoid sun exposure during hot
    weather.
14. Answer: C
    Global aphasia occurs when all language functions are affected. Receptive aphasia, also
    known as Wernicke’s aphasia, affects the ability to comprehend written or spoken words.
    Expressive aphasia, also known as Broca’s aphasia, affected the patient’s ability to form
    language and express thoughts. Conduction aphasia refers to abnormalities in speech
    repetition.
15. Answer: D
    Patients with a history of headaches, especially migraines, should be taught to keep a
    food diary to identify potential food triggers. Typical headache triggers include alcohol,
    aged cheeses, processed meats, and chocolate and caffeine-containing products.
16. Answer: B
    An explosive headache or “the worst headache I’ve ever had” is typically the first
    presenting symptom of a bleeding cerebral aneurysm. Photophobia, seizures, and
    hemiparesis may occur later.
17. Answer: C
    An embolic injury, caused by a traveling clot, may result from atrial fibrillation. Blood
    may pool in the fibrillating atrium and be released to travel up the cerebral artery to the
    brain. Persistent hypertension may place the patient at risk for a thrombotic injury to the
    brain. Subarachnoid hemorrhage and skull fractures aren’t associated with emboli.
18. Answer: D
    Women with spinal cord injuries who were sexually active may continue having sexual
    intercourse and must be reminded that they can still become pregnant. She may be fully
    capable of achieving orgasm. An indwelling urinary catheter may be left in place during
    sexual intercourse. Positioning will need to be adjusted to fit the patient’s needs.
19. Answer: A
    The nurse must have a wrench taped on the vest at all times for quick halo removal in
    emergent situations. The brace isn’t to be removed for any other reason until the cervical
    fracture is healed. Placing a pillow under the patient’s head may alter the stability of the
    brace.
20. Answer: D
    Nimodipine is a calcium channel blocker that acts on cerebral blood vessels to reduce
    vasospasm. The drug doesn’t increase the amount of calcium, affect cerebral vasculature
    growth, or reduce cerebral oxygen demand.
21. Answer: D
    Men with spinal cord injury should be taught that the higher the level of the lesion, the
    better their sexual function will be. The sacral region is the lowest area on the spinal
    column and injury to this area will cause more erectile dysfunction.
22. Answer: C
    Tegretol should be taken with food to minimize GI distress. Taking it at meals will also
    establish a regular routine, which should help compliance.
23. Answer: B
    Patient with pneumococcal meningitis require respiratory isolation for the first 24 hours
    after treatment is initiated.
24. Answer: A
    Early symptoms of ALS include fatigue while talking, dysphagia, and weakness of the
    hands and arms. ALS doesn’t cause a change in mental status, paresthesia, or fractures.
25. Answer: C
    Increased pressure within the portal veins causes them to bulge, leading to rupture and
    bleeding into the lower esophagus. Bleeding associated with esophageal varices doesn’t
    stem from esophageal perforation, pulmonary hypertension, or peptic ulcers.
26. Answer: B
    The nurse should wear gloves and a gown when removing the patient’s bedpan because
    the type A hepatitis virus occurs in stools. It may also occur in blood, nasotracheal
    secretions, and urine. Type A hepatitis isn’t transmitted through the air by way of
    droplets. Special precautions aren’t needed when feeding the patient, but disposable
    utensils should be used.
27. Answer: B
    The patient should notify the doctor if he has difficulty inserting the irrigation tube into
    the stoma. Difficulty with insertion may indicate stenosis of the bowel. Abdominal
    cramping and expulsion of flatus may normally occur with irrigation. The procedure will
    often take an hour to complete.
28. Answer: B
    An indwelling urinary catheter is kept in place several days after this surgery to prevent
    urine retention that could place pressure on the perineal wound. An indwelling urinary
    catheter may be a source of postoperative urinary tract infection. Urine won’t
    contaminate the wound. An indwelling urinary catheter won’t necessarily show bladder
    trauma.
29. Answer: D
    The colostomy may not function for 2 days or more (48 to 72 hours) after surgery.
    Therefore, the normal plan of care can be followed. Since no fecal drainage is expected
    for 48 to 72 hours after a colostomy (only mucous and serosanguineous), the doctor
    doesn’t have to be notified and the stoma shouldn’t be irrigated at this time.
30. Answer: C
    If the patient’s GI tract is functioning, enteral nutrition via a feeding tube is the preferred
    method. Peripheral and total parenteral nutrition places the patient at risk for infection. If
    the patient is unable to consume foods by mouth, oral liquid supplements are
    contraindicated.
31. Answer: A
    A hypertonic solution causes water to shift from the cells into the plasma because the
    hypertonic solution has a greater osmotic pressure than the cells. A hypotonic solution
    has a lower osmotic pressure than that of the cells. It causes fluid to shift into the cells,
    possibly resulting in rupture. An isotonic solution, which has the same osmotic pressure
    as the cells, wouldn’t cause any shift. A solution’s alkalinity is related to the hydrogen
    ion concentration, not its osmotic effect.
32. Answer: C
    Particles move from an area of greater osmolarity to one of lesser osmolarity through
diffusion. Active transport is the movement of particles though energy expenditure from
    other sources such as enzymes. Osmosis is the movement of a pure solvent through a
    semipermeable membrane from an area of greater osmolarity to one of lesser osmolarity
    until equalization occurs. The membrane is impermeable to the solute but permeable to
    the solvent. Filtration is the process by which fluid is forced through a membrane by a
    difference in pressure; small molecules pass through, but large ones don’t.
33. Answer: A
    Tenting of chest skin when pinched indicates decreased skin elasticity due to
    dehydration. Hand veins fill slowly with dehydration, not rapidly. A pulse that isn’t easily
    obliterated and neck vein distention indicate fluid overload, not dehydration.
34. Answer: C
    Administering a tap water enema until return is clear would most likely contribute to a
    hypo-osmolar state. Because tap water is hypotonic, it would be absorbed by the body,
    diluting the body fluid concentration and lowering osmolarity. Weighing the patient is the
    easiest, most accurate method to determine fluid changes. Therefore, it helps identify
    rather than contribute to fluid imbalance. Nasogastric tube irrigation with normal saline
    solution wouldn’t cause a shift in fluid balance. Drinking broth wouldn’t contribute to a
    hypo-osmolar state because it doesn’t replace sodium and water lost through excessive
    perspiration.
35. Answer: D
    An orthostatic blood pressure indicates an extracellular fluid volume deficit. (The
    extracellular compartment consists of both the intravascular compartment and interstitial
    space.) A fluid volume deficit within the intravascular compartment would cause
    tachycardia, not bradycardia or orthostatic blood pressure change. A central venous
    pressure of 6 mm Hg is in the high normal range, indicating adequate hydration. Pitting
    edema indicates fluid volume overload.
36. Answer: C
    The respiratory and renal systems act as compensatory mechanisms to counteract-base
    imbalances. The lungs alter the carbon dioxide levels in the blood by increasing or
    decreasing the rate and depth of respirations, thereby increasing or decreasing carbon
    dioxide elimination. The liver, pancreas, and heart play no part in compensating for acid-
    base imbalances.
37. Answer: B
    Anuria refers to a urine output of less than 100 ml in 24 hours. The baseline for urine
    output and renal function is 30 ml of urine per hour. A urine output of at least 100 ml in 2
    hours is within normal limits. Voiding at night is called nocturia. Pain and burning on
    urination is called dysuria.
38. Answer: A
    To obtain urine properly, the nurse should aspirate it from a port, using a sterile syringe
    after cleaning the port. Opening a closed urine drainage system increases the risk of
    urinary tract infection. Standard precautions specify the use of gloves during contract
    with body fluids; however, sterile gloves aren’t necessary.
39. Answer: C
    Although postoperative pain is expected, the nurse should ensure that other factors, such
    as an obstructed irrigation catheter, aren’t the cause of the pain. After assessing catheter
    patency, the nurse should administer an analgesic such as meperidine as prescribed.
Increasing the I.V. flow rate may worse the pain. Notifying the doctor isn’t necessary
    unless the pain is severe or unrelieved by the prescribed medication.
40. Answer: A
    Renal calculi most commonly from in the kidney. They may remain there or become
    lodged anywhere along the urinary tract. The ureter, bladder, and urethra are less
    common sites of renal calculi formation.
41. Answer: A
    Ureterolithiasis typically causes such acute, severe pain that the patient can’t rest and
    becomes increasingly anxious. Therefore, the nursing diagnosis of pain takes highest
    priority. Risk for infection and altered urinary elimination are appropriate once the
    patient’s pain is controlled. Altered nutrition: less than body requirements isn’t
    appropriate at this time.
42. Answer: B
    Normal urine pH is 4.5 to 8; therefore, a urine pH of 3 is abnormal and may indicate such
    conditions as renal tuberculosis, pyrexia, phenylketonuria, alkaptonuria, and acidosis.
    Urine specific gravity normally ranges from 1.002 to 1.032, making the patient’s value
    normal. Normally, urine contains no protein, glucose, ketones, bilirubin, bacteria, casts,
    or crystals.
43. Answer: C
    The nephron is the kidney’s functioning unit. The glomerulus, Bowman’s capsule, and
    tubular system are components of the nephron.
44. Answer: A
    As a response to shock, the renin-angiotensin-aldosterone system alters renal function by
    decreasing urine output and increasing reabsorption of sodium and water. Reduced renal
    perfusion stimulates the renin-angiotensin-aldosterone system in an effort to conserve
    circulating volume.
45. Answer: D
    The bladder isn’t usually palpable unless it is distended. The feeling of pressure is usually
    relieved with urination. Reduced bladder tone due to general anesthesia is a common
    postoperative complication that causes difficulty in voiding. A urinary tract infection and
    renal calculi aren’t palpable. The kidneys aren’t palpable above the symphysis pubis.
46. Answer: A
    A pericardial friction rub may be present with the pericardial effusion of pericarditis. The
    lungs are typically clear when auscultated. Sitting up and leaning forward often relieves
    pericarditis pain. An S3 indicates left-sided heart failure and isn’t usually present with
    pericarditis.
47. Answer: D
    Assessing jugular vein distention should be done when the patient is in semi-Fowler’s
    position (head of the bed elevated 30 to 45 degrees). If the patient lies flat, the veins will
    be more distended; if he sits upright, the veins will be flat.
48. Answer: C
    Smoking should receive highest priority when trying to reduce risk factors for with
    respiratory complications. Losing weight and decreasing salt and caffeine intake can help
    to decrease risk factors for hypertension.
49. Answer: B
    The correct ratio of compressions to ventilations when one rescuer performs CPR is 15:2
50. Answer: D
    The lungs and kidneys are the body’s regulators of homeostasis. The lungs are
    responsible for removing fluid and carbon dioxide; the kidneys maintain a balance of
    fluid and electrolytes. The other organs play secondary roles in maintaining homeostasis.




   PSYCHIATRIC NSG

   1. Which of the following medications would the nurse in-charge expect the doctor to
      order to reverse a dystonic reaction?

       a. Procholorperazine (Compazine)

       b. Diphenhydramine (Benadryl)

       c. Haloperidol (Haldol)

       d. Midazolam (Versed)

       2. While pacing in the hall, a female patient with paranoid schizophrenia runs to the
       nurse and says, “Why are you poisoning me? I know you work for central thought
       control! You can keep my thoughts. Give me back my soul!” how should the nurse
       respond?

       a. “I’m a nurse, I’m not poisoning you. It’s against the nursing code of ethics.”

       b. “I’m a nurse, and you’re a patient in the hospital. I’m not going to harm you.”

       c. “I’m not poisoning you. And how could I possibly steal your soul?”

       d. “I sense anger, Are you feeling angry today?”

       3. After completing chemical detoxification and a 12-step program to treat crack
       addiction, a male patient is being prepared for discharge. Which remark by the patient
       indicates a realistic view of the future?

       a. “I’m never going to use crack again.”

       b. “I know what I have to do. I have to limit my crack use.”

       c. “I’m going to take 1 day at a time. I’m not making any promises.”

       d. “I can’t touch crack again, but I sure could use a drink. I’ve earned it.”
4. The nurse formulates a nursing diagnosis of “impaired verbal communication” fora
male patient with schizotypal personality disorder. Based on this nursing diagnosis,
which nursing intervention is most appropriate?

a. Helping the patient to participate in social interactions

b. Establishing a one-on-one relationship with the patient

c. Establishing alternative forms of communication

d. Allowing the patient to decide when he wants to participate in verbal
communication with you

5. A female patient with obsessive-compulsive disorder tells the nurse that he must
check the lock on his apartment door 25 times before leaving for an appointment. The
nurse knows that this behavior represents the patient’s attempt to:

a. Call attention to himself

b. Control his thoughts

c. Maintain the safety of his home

d. Reduce anxiety

6. A patient, age 42, is admitted for surgical biopsy of a suspicious lump in her left
breast. When the nurse comes to her surgery, she is tearfully finishing a letter to her
children. She tells the nurse, “I want to leave this for my children in case anything
goes wrong today. “Which response by the nurse would be most therapeutic?

a. “In case anything goes wrong? What are your thoughts and feelings right now?”

b. “I can’t understand that you’re nervous, but this is really a minor procedure. You’ll
be back in your room before you know it.”

c. “Try to take a few deep breaths and relax. I have some medication that will help.”

d. “I’m sure your children know how much you love them. You’ll be able to talk to
them on the phone in a few hours.”

7. Which nursing intervention is most important when restraining a violent male
patient?

a. Reviewing hospital policy regarding how long the patient can be restrained

b. Preparing a p.r.n. dose of the patient’s psychotropic medication
c. Checking that the restraints have been applied correctly

d. Asking if the patient needs to use the bathroom or is thirsty

8. How soon after chlorpromazine administration should the nurse in charge expect to
see a patient’s delusion thoughts and hallucinations eliminated?

a. Several minutes

b. Several hours

c. Several days

d. Several weeks

9. Mental health laws in each state specify when restraints can be used and which
type of restraints are allowed. Most laws stipulate that restraints can be used:

a. For a maximum of 2 hours

b. As necessary to control the patient

c. If the patient poses a present danger to self or others

d. Only with the patient’s consent

10. A female patient has been severely depressed since her husband died 6 months
ago. Her doctor prescribes amitriptyline hydrochloride (Elavil), 50 mg P.O. daily.
Before administering amitriptyline, the nurse reviews the patient’s medical history.
Which preexisting condition would require cautions use of this drug?

a. Hiatal hernia

b. Hypernatremia

c. Hepatic disease

d. Hypokalemia

11. The physician orders a new medication for a male client with generalized anxiety
disorder. During medication teaching, which statement or question by the nurse
would be most appropriate?

a. “Take this medication. It will reduce your anxiety.”

b. “Do you have any concern about taking the medication?”
MCQs for Entrance Test for BN, MN, MSN Nursing by RS MEHTA
MCQs for Entrance Test for BN, MN, MSN Nursing by RS MEHTA
MCQs for Entrance Test for BN, MN, MSN Nursing by RS MEHTA
MCQs for Entrance Test for BN, MN, MSN Nursing by RS MEHTA
MCQs for Entrance Test for BN, MN, MSN Nursing by RS MEHTA
MCQs for Entrance Test for BN, MN, MSN Nursing by RS MEHTA
MCQs for Entrance Test for BN, MN, MSN Nursing by RS MEHTA
MCQs for Entrance Test for BN, MN, MSN Nursing by RS MEHTA
MCQs for Entrance Test for BN, MN, MSN Nursing by RS MEHTA
MCQs for Entrance Test for BN, MN, MSN Nursing by RS MEHTA
MCQs for Entrance Test for BN, MN, MSN Nursing by RS MEHTA
MCQs for Entrance Test for BN, MN, MSN Nursing by RS MEHTA
MCQs for Entrance Test for BN, MN, MSN Nursing by RS MEHTA
MCQs for Entrance Test for BN, MN, MSN Nursing by RS MEHTA
MCQs for Entrance Test for BN, MN, MSN Nursing by RS MEHTA
MCQs for Entrance Test for BN, MN, MSN Nursing by RS MEHTA
MCQs for Entrance Test for BN, MN, MSN Nursing by RS MEHTA
MCQs for Entrance Test for BN, MN, MSN Nursing by RS MEHTA
MCQs for Entrance Test for BN, MN, MSN Nursing by RS MEHTA
MCQs for Entrance Test for BN, MN, MSN Nursing by RS MEHTA
MCQs for Entrance Test for BN, MN, MSN Nursing by RS MEHTA

Mais conteúdo relacionado

Mais procurados

Emergency nursing questionnaires
Emergency nursing questionnairesEmergency nursing questionnaires
Emergency nursing questionnairesJoan Delgado
 
Questions and answers
Questions and answersQuestions and answers
Questions and answersWency Niez
 
Fundamentals of nursing practice exam
Fundamentals of nursing practice examFundamentals of nursing practice exam
Fundamentals of nursing practice examNursing Path
 
Nursing education quiz
Nursing education quizNursing education quiz
Nursing education quizNursing Path
 
Fundamental of nursing practice exam 4
Fundamental of nursing practice exam 4Fundamental of nursing practice exam 4
Fundamental of nursing practice exam 4Nursing Path
 
Physical assessment exam study guide
Physical assessment exam study guidePhysical assessment exam study guide
Physical assessment exam study guideaamir khan
 
The ICN Code of Ethics for Nurses
The ICN Code of Ethics for NursesThe ICN Code of Ethics for Nurses
The ICN Code of Ethics for NursesProf Vijayraddi
 
Gordons 11-functional-health-patterns
Gordons 11-functional-health-patternsGordons 11-functional-health-patterns
Gordons 11-functional-health-patternsReihchelle Bayad
 
Nursing diagnosis
Nursing diagnosisNursing diagnosis
Nursing diagnosisArifa T N
 
DOCUMENTATION IN NURSING
DOCUMENTATION IN NURSINGDOCUMENTATION IN NURSING
DOCUMENTATION IN NURSINGANILKUMAR BR
 
Physical assessment quiz dr. khaled khader
Physical assessment quiz   dr. khaled khaderPhysical assessment quiz   dr. khaled khader
Physical assessment quiz dr. khaled khaderkhaledkhader
 
Fundamental of Nursing 1st semester BSN mcqs for practice,Educational Platfor...
Fundamental of Nursing 1st semester BSN mcqs for practice,Educational Platfor...Fundamental of Nursing 1st semester BSN mcqs for practice,Educational Platfor...
Fundamental of Nursing 1st semester BSN mcqs for practice,Educational Platfor...MuhammadRazaBuzdar
 
Nursing care plan ppt final draft
Nursing care plan ppt final draftNursing care plan ppt final draft
Nursing care plan ppt final draftgntc
 
Communication in nursing
Communication in nursingCommunication in nursing
Communication in nursingNursing Path
 
Medical surgical nursing quiz
Medical surgical nursing quizMedical surgical nursing quiz
Medical surgical nursing quizNursing Path
 
Nursing Assessment
Nursing AssessmentNursing Assessment
Nursing Assessmentpreet kaur
 

Mais procurados (20)

Drill 9
Drill 9Drill 9
Drill 9
 
Emergency nursing questionnaires
Emergency nursing questionnairesEmergency nursing questionnaires
Emergency nursing questionnaires
 
Questions and answers
Questions and answersQuestions and answers
Questions and answers
 
Fundamentals of nursing practice exam
Fundamentals of nursing practice examFundamentals of nursing practice exam
Fundamentals of nursing practice exam
 
Ethics in nursing
Ethics in nursingEthics in nursing
Ethics in nursing
 
Nursing education quiz
Nursing education quizNursing education quiz
Nursing education quiz
 
Fundamental of nursing practice exam 4
Fundamental of nursing practice exam 4Fundamental of nursing practice exam 4
Fundamental of nursing practice exam 4
 
Physical assessment exam study guide
Physical assessment exam study guidePhysical assessment exam study guide
Physical assessment exam study guide
 
The ICN Code of Ethics for Nurses
The ICN Code of Ethics for NursesThe ICN Code of Ethics for Nurses
The ICN Code of Ethics for Nurses
 
Gordons 11-functional-health-patterns
Gordons 11-functional-health-patternsGordons 11-functional-health-patterns
Gordons 11-functional-health-patterns
 
Nursing diagnosis
Nursing diagnosisNursing diagnosis
Nursing diagnosis
 
DOCUMENTATION IN NURSING
DOCUMENTATION IN NURSINGDOCUMENTATION IN NURSING
DOCUMENTATION IN NURSING
 
Physical assessment quiz dr. khaled khader
Physical assessment quiz   dr. khaled khaderPhysical assessment quiz   dr. khaled khader
Physical assessment quiz dr. khaled khader
 
Fundamental of Nursing 1st semester BSN mcqs for practice,Educational Platfor...
Fundamental of Nursing 1st semester BSN mcqs for practice,Educational Platfor...Fundamental of Nursing 1st semester BSN mcqs for practice,Educational Platfor...
Fundamental of Nursing 1st semester BSN mcqs for practice,Educational Platfor...
 
Nursing care plan ppt final draft
Nursing care plan ppt final draftNursing care plan ppt final draft
Nursing care plan ppt final draft
 
Nursing theories-ppt
Nursing theories-pptNursing theories-ppt
Nursing theories-ppt
 
Communication in nursing
Communication in nursingCommunication in nursing
Communication in nursing
 
Health promotion model
Health promotion modelHealth promotion model
Health promotion model
 
Medical surgical nursing quiz
Medical surgical nursing quizMedical surgical nursing quiz
Medical surgical nursing quiz
 
Nursing Assessment
Nursing AssessmentNursing Assessment
Nursing Assessment
 

Destaque

Nursing question practice case
Nursing question practice caseNursing question practice case
Nursing question practice casesarosem
 
Nursing research quiz series
Nursing research quiz seriesNursing research quiz series
Nursing research quiz seriesNursing Path
 
Nursing management quiz
Nursing management quizNursing management quiz
Nursing management quizNursing Path
 
Multiple choice questions with answers
Multiple choice questions with answersMultiple choice questions with answers
Multiple choice questions with answersClassic Tech
 

Destaque (9)

Nursing question practice case
Nursing question practice caseNursing question practice case
Nursing question practice case
 
Nursing research quiz series
Nursing research quiz seriesNursing research quiz series
Nursing research quiz series
 
Nursing management quiz
Nursing management quizNursing management quiz
Nursing management quiz
 
Genito Urinary Nursing
Genito Urinary NursingGenito Urinary Nursing
Genito Urinary Nursing
 
Musculoskeletal Nursing
Musculoskeletal NursingMusculoskeletal Nursing
Musculoskeletal Nursing
 
Respiratory Nursing
Respiratory NursingRespiratory Nursing
Respiratory Nursing
 
Gastrointestinal Nursing
Gastrointestinal NursingGastrointestinal Nursing
Gastrointestinal Nursing
 
Multiple choice questions with answers
Multiple choice questions with answersMultiple choice questions with answers
Multiple choice questions with answers
 
Dec 2012 NLE TIPS MS (A)
Dec 2012 NLE TIPS MS (A)Dec 2012 NLE TIPS MS (A)
Dec 2012 NLE TIPS MS (A)
 

Semelhante a MCQs for Entrance Test for BN, MN, MSN Nursing by RS MEHTA

maternal child health nursing MCQ 2.docx
maternal child health nursing MCQ 2.docxmaternal child health nursing MCQ 2.docx
maternal child health nursing MCQ 2.docxMarieBagunu
 
K.K-MIDWIFERY MCQS REVISION QUESTION.pptx
K.K-MIDWIFERY MCQS REVISION QUESTION.pptxK.K-MIDWIFERY MCQS REVISION QUESTION.pptx
K.K-MIDWIFERY MCQS REVISION QUESTION.pptxKelvinkebu
 
2016 Licensing exam-1.docx
2016 Licensing exam-1.docx2016 Licensing exam-1.docx
2016 Licensing exam-1.docxRBGroup
 
Obs and gynae data bank
Obs and gynae data bankObs and gynae data bank
Obs and gynae data bankwiseman chanda
 
When assessing the adequacy of sperm for conception to occur
When assessing the adequacy of sperm for conception to occurWhen assessing the adequacy of sperm for conception to occur
When assessing the adequacy of sperm for conception to occurartqlison
 
1. A nurse responds to the cardiac monitor alarm of a patient an.docx
1. A nurse responds to the cardiac monitor alarm of a patient an.docx1. A nurse responds to the cardiac monitor alarm of a patient an.docx
1. A nurse responds to the cardiac monitor alarm of a patient an.docxmonicafrancis71118
 
8. cesarean section
8. cesarean section8. cesarean section
8. cesarean sectionHishgeeubuns
 
اسئلة تمريض 1
اسئلة تمريض 1اسئلة تمريض 1
اسئلة تمريض 1al asheery
 
How to have a good study habits 2
How to have a good study habits 2How to have a good study habits 2
How to have a good study habits 2shenell delfin
 
Gestational diabetes case study 2nd one
Gestational diabetes case study 2nd oneGestational diabetes case study 2nd one
Gestational diabetes case study 2nd oneLisette Allender
 
recall Jan 2023.pptx
recall Jan 2023.pptxrecall Jan 2023.pptx
recall Jan 2023.pptxAishaNajeeb1
 
Communicable disease ans. key
Communicable disease ans. keyCommunicable disease ans. key
Communicable disease ans. keyFern Mosquera
 
OBG%20FMGE%20LRR%20Part%202.pdf
OBG%20FMGE%20LRR%20Part%202.pdfOBG%20FMGE%20LRR%20Part%202.pdf
OBG%20FMGE%20LRR%20Part%202.pdfpratappankaj2017
 
Krok 2 - 2013 (Gynecology)
Krok 2 - 2013 (Gynecology)Krok 2 - 2013 (Gynecology)
Krok 2 - 2013 (Gynecology)Eneutron
 
Gyn obs final exam, 2014
Gyn obs final exam, 2014Gyn obs final exam, 2014
Gyn obs final exam, 2014Tadiwos Gintamo
 

Semelhante a MCQs for Entrance Test for BN, MN, MSN Nursing by RS MEHTA (20)

Pre board mcn
Pre board mcnPre board mcn
Pre board mcn
 
maternal child health nursing MCQ 2.docx
maternal child health nursing MCQ 2.docxmaternal child health nursing MCQ 2.docx
maternal child health nursing MCQ 2.docx
 
Comprhensive
ComprhensiveComprhensive
Comprhensive
 
K.K-MIDWIFERY MCQS REVISION QUESTION.pptx
K.K-MIDWIFERY MCQS REVISION QUESTION.pptxK.K-MIDWIFERY MCQS REVISION QUESTION.pptx
K.K-MIDWIFERY MCQS REVISION QUESTION.pptx
 
2016 Licensing exam-1.docx
2016 Licensing exam-1.docx2016 Licensing exam-1.docx
2016 Licensing exam-1.docx
 
Obs and gynae data bank
Obs and gynae data bankObs and gynae data bank
Obs and gynae data bank
 
1
11
1
 
When assessing the adequacy of sperm for conception to occur
When assessing the adequacy of sperm for conception to occurWhen assessing the adequacy of sperm for conception to occur
When assessing the adequacy of sperm for conception to occur
 
1. A nurse responds to the cardiac monitor alarm of a patient an.docx
1. A nurse responds to the cardiac monitor alarm of a patient an.docx1. A nurse responds to the cardiac monitor alarm of a patient an.docx
1. A nurse responds to the cardiac monitor alarm of a patient an.docx
 
Cesarian section
Cesarian sectionCesarian section
Cesarian section
 
8. cesarean section
8. cesarean section8. cesarean section
8. cesarean section
 
اسئلة تمريض 1
اسئلة تمريض 1اسئلة تمريض 1
اسئلة تمريض 1
 
How to have a good study habits 2
How to have a good study habits 2How to have a good study habits 2
How to have a good study habits 2
 
Gestational diabetes case study 2nd one
Gestational diabetes case study 2nd oneGestational diabetes case study 2nd one
Gestational diabetes case study 2nd one
 
recall Jan 2023.pptx
recall Jan 2023.pptxrecall Jan 2023.pptx
recall Jan 2023.pptx
 
Communicable disease ans. key
Communicable disease ans. keyCommunicable disease ans. key
Communicable disease ans. key
 
OBG%20FMGE%20LRR%20Part%202.pdf
OBG%20FMGE%20LRR%20Part%202.pdfOBG%20FMGE%20LRR%20Part%202.pdf
OBG%20FMGE%20LRR%20Part%202.pdf
 
Krok 2 - 2013 (Gynecology)
Krok 2 - 2013 (Gynecology)Krok 2 - 2013 (Gynecology)
Krok 2 - 2013 (Gynecology)
 
Gyn obs final exam, 2014
Gyn obs final exam, 2014Gyn obs final exam, 2014
Gyn obs final exam, 2014
 
Drill 3
Drill 3Drill 3
Drill 3
 

Mais de BP KOIRALA INSTITUTE OF HELATH SCIENCS,, NEPAL

Mais de BP KOIRALA INSTITUTE OF HELATH SCIENCS,, NEPAL (20)

Paradigm shift in nursing research by RS MEHTA
Paradigm shift in nursing research by RS MEHTAParadigm shift in nursing research by RS MEHTA
Paradigm shift in nursing research by RS MEHTA
 
Jiwani of RS Mehta book.pdf
Jiwani of RS Mehta book.pdfJiwani of RS Mehta book.pdf
Jiwani of RS Mehta book.pdf
 
Ph.D. Thesis on HBC by RS Mehta.pdf
Ph.D. Thesis on HBC by RS Mehta.pdfPh.D. Thesis on HBC by RS Mehta.pdf
Ph.D. Thesis on HBC by RS Mehta.pdf
 
M. Sc. Nursing Thesis by RS Mehta.pdf
M. Sc. Nursing Thesis  by RS Mehta.pdfM. Sc. Nursing Thesis  by RS Mehta.pdf
M. Sc. Nursing Thesis by RS Mehta.pdf
 
Ph.D. Thesis on HBC by RS Mehta.pdf
Ph.D. Thesis on HBC by RS Mehta.pdfPh.D. Thesis on HBC by RS Mehta.pdf
Ph.D. Thesis on HBC by RS Mehta.pdf
 
bsc pancreatitis 8.pptx
bsc pancreatitis 8.pptxbsc pancreatitis 8.pptx
bsc pancreatitis 8.pptx
 
12-lead EKG Interpretation1.pdf
12-lead EKG Interpretation1.pdf12-lead EKG Interpretation1.pdf
12-lead EKG Interpretation1.pdf
 
4. Advocacy in Nursing.pdf
4. Advocacy in Nursing.pdf4. Advocacy in Nursing.pdf
4. Advocacy in Nursing.pdf
 
3. Legal Aspects in Nursing.pdf
3. Legal Aspects in Nursing.pdf3. Legal Aspects in Nursing.pdf
3. Legal Aspects in Nursing.pdf
 
1. Ethics and Values.pdf
1. Ethics and Values.pdf1. Ethics and Values.pdf
1. Ethics and Values.pdf
 
2. ICN Code for Nursing Ethics.pdf
2. ICN Code for Nursing Ethics.pdf2. ICN Code for Nursing Ethics.pdf
2. ICN Code for Nursing Ethics.pdf
 
RS MEHTA Photos 24 yrs in BPKIHS.ppsx
RS MEHTA Photos 24 yrs in BPKIHS.ppsxRS MEHTA Photos 24 yrs in BPKIHS.ppsx
RS MEHTA Photos 24 yrs in BPKIHS.ppsx
 
9. Experiences of Singapore CGH.ppsx
9. Experiences of Singapore CGH.ppsx9. Experiences of Singapore CGH.ppsx
9. Experiences of Singapore CGH.ppsx
 
International Visit by RS MEHTA.ppsx
International  Visit by RS MEHTA.ppsxInternational  Visit by RS MEHTA.ppsx
International Visit by RS MEHTA.ppsx
 
Ram Sharan Mehta Jiwani
Ram Sharan Mehta Jiwani Ram Sharan Mehta Jiwani
Ram Sharan Mehta Jiwani
 
4. advocacy in nursing
4. advocacy in nursing4. advocacy in nursing
4. advocacy in nursing
 
3. legal aspects in nursing
3. legal aspects in nursing3. legal aspects in nursing
3. legal aspects in nursing
 
2. icn code for nursing ethics
2. icn code for nursing ethics2. icn code for nursing ethics
2. icn code for nursing ethics
 
1. ethics and values
1. ethics and values1. ethics and values
1. ethics and values
 
CV Ram Sharan Mehta
CV Ram Sharan MehtaCV Ram Sharan Mehta
CV Ram Sharan Mehta
 

Último

Introduction to Political Parties (1).ppt
Introduction to Political Parties (1).pptIntroduction to Political Parties (1).ppt
Introduction to Political Parties (1).pptSohamChavan9
 
Application deck- Cyril Caudroy-2024.pdf
Application deck- Cyril Caudroy-2024.pdfApplication deck- Cyril Caudroy-2024.pdf
Application deck- Cyril Caudroy-2024.pdfCyril CAUDROY
 
原版定制copy澳洲查尔斯达尔文大学毕业证CDU毕业证成绩单留信学历认证保障质量
原版定制copy澳洲查尔斯达尔文大学毕业证CDU毕业证成绩单留信学历认证保障质量原版定制copy澳洲查尔斯达尔文大学毕业证CDU毕业证成绩单留信学历认证保障质量
原版定制copy澳洲查尔斯达尔文大学毕业证CDU毕业证成绩单留信学历认证保障质量sehgh15heh
 
格里菲斯大学毕业证(Griffith毕业证)#文凭成绩单#真实留信学历认证永久存档
格里菲斯大学毕业证(Griffith毕业证)#文凭成绩单#真实留信学历认证永久存档格里菲斯大学毕业证(Griffith毕业证)#文凭成绩单#真实留信学历认证永久存档
格里菲斯大学毕业证(Griffith毕业证)#文凭成绩单#真实留信学历认证永久存档208367051
 
ME 205- Chapter 6 - Pure Bending of Beams.pdf
ME 205- Chapter 6 - Pure Bending of Beams.pdfME 205- Chapter 6 - Pure Bending of Beams.pdf
ME 205- Chapter 6 - Pure Bending of Beams.pdfaae4149584
 
办理哈珀亚当斯大学学院毕业证书文凭学位证书
办理哈珀亚当斯大学学院毕业证书文凭学位证书办理哈珀亚当斯大学学院毕业证书文凭学位证书
办理哈珀亚当斯大学学院毕业证书文凭学位证书saphesg8
 
办理学位证(Massey证书)新西兰梅西大学毕业证成绩单原版一比一
办理学位证(Massey证书)新西兰梅西大学毕业证成绩单原版一比一办理学位证(Massey证书)新西兰梅西大学毕业证成绩单原版一比一
办理学位证(Massey证书)新西兰梅西大学毕业证成绩单原版一比一A SSS
 
定制英国克兰菲尔德大学毕业证成绩单原版一比一
定制英国克兰菲尔德大学毕业证成绩单原版一比一定制英国克兰菲尔德大学毕业证成绩单原版一比一
定制英国克兰菲尔德大学毕业证成绩单原版一比一z zzz
 
办理学位证(纽伦堡大学文凭证书)纽伦堡大学毕业证成绩单原版一模一样
办理学位证(纽伦堡大学文凭证书)纽伦堡大学毕业证成绩单原版一模一样办理学位证(纽伦堡大学文凭证书)纽伦堡大学毕业证成绩单原版一模一样
办理学位证(纽伦堡大学文凭证书)纽伦堡大学毕业证成绩单原版一模一样umasea
 
原版定制卡尔加里大学毕业证(UC毕业证)留信学历认证
原版定制卡尔加里大学毕业证(UC毕业证)留信学历认证原版定制卡尔加里大学毕业证(UC毕业证)留信学历认证
原版定制卡尔加里大学毕业证(UC毕业证)留信学历认证diploma001
 
Ioannis Tzachristas Self-Presentation for MBA.pdf
Ioannis Tzachristas Self-Presentation for MBA.pdfIoannis Tzachristas Self-Presentation for MBA.pdf
Ioannis Tzachristas Self-Presentation for MBA.pdfjtzach
 
Storytelling, Ethics and Workflow in Documentary Photography
Storytelling, Ethics and Workflow in Documentary PhotographyStorytelling, Ethics and Workflow in Documentary Photography
Storytelling, Ethics and Workflow in Documentary PhotographyOrtega Alikwe
 
Digital Marketing Training Institute in Mohali, India
Digital Marketing Training Institute in Mohali, IndiaDigital Marketing Training Institute in Mohali, India
Digital Marketing Training Institute in Mohali, IndiaDigital Discovery Institute
 
Crack JAG. Guidance program for entry to JAG Dept. & SSB interview
Crack JAG. Guidance program for entry to JAG Dept. & SSB interviewCrack JAG. Guidance program for entry to JAG Dept. & SSB interview
Crack JAG. Guidance program for entry to JAG Dept. & SSB interviewNilendra Kumar
 
Unlock Your Creative Potential: 7 Skills for Content Creator Evolution
Unlock Your Creative Potential: 7 Skills for Content Creator EvolutionUnlock Your Creative Potential: 7 Skills for Content Creator Evolution
Unlock Your Creative Potential: 7 Skills for Content Creator EvolutionRhazes Ghaisan
 
定制(UQ毕业证书)澳洲昆士兰大学毕业证成绩单原版一比一
定制(UQ毕业证书)澳洲昆士兰大学毕业证成绩单原版一比一定制(UQ毕业证书)澳洲昆士兰大学毕业证成绩单原版一比一
定制(UQ毕业证书)澳洲昆士兰大学毕业证成绩单原版一比一lvtagr7
 
Black and White Minimalist Co Letter.pdf
Black and White Minimalist Co Letter.pdfBlack and White Minimalist Co Letter.pdf
Black and White Minimalist Co Letter.pdfpadillaangelina0023
 
办理(Salford毕业证书)索尔福德大学毕业证成绩单原版一比一
办理(Salford毕业证书)索尔福德大学毕业证成绩单原版一比一办理(Salford毕业证书)索尔福德大学毕业证成绩单原版一比一
办理(Salford毕业证书)索尔福德大学毕业证成绩单原版一比一diploma 1
 
LinkedIn for Your Job Search in April 2024
LinkedIn for Your Job Search in April 2024LinkedIn for Your Job Search in April 2024
LinkedIn for Your Job Search in April 2024Bruce Bennett
 

Último (20)

Introduction to Political Parties (1).ppt
Introduction to Political Parties (1).pptIntroduction to Political Parties (1).ppt
Introduction to Political Parties (1).ppt
 
Application deck- Cyril Caudroy-2024.pdf
Application deck- Cyril Caudroy-2024.pdfApplication deck- Cyril Caudroy-2024.pdf
Application deck- Cyril Caudroy-2024.pdf
 
原版定制copy澳洲查尔斯达尔文大学毕业证CDU毕业证成绩单留信学历认证保障质量
原版定制copy澳洲查尔斯达尔文大学毕业证CDU毕业证成绩单留信学历认证保障质量原版定制copy澳洲查尔斯达尔文大学毕业证CDU毕业证成绩单留信学历认证保障质量
原版定制copy澳洲查尔斯达尔文大学毕业证CDU毕业证成绩单留信学历认证保障质量
 
格里菲斯大学毕业证(Griffith毕业证)#文凭成绩单#真实留信学历认证永久存档
格里菲斯大学毕业证(Griffith毕业证)#文凭成绩单#真实留信学历认证永久存档格里菲斯大学毕业证(Griffith毕业证)#文凭成绩单#真实留信学历认证永久存档
格里菲斯大学毕业证(Griffith毕业证)#文凭成绩单#真实留信学历认证永久存档
 
ME 205- Chapter 6 - Pure Bending of Beams.pdf
ME 205- Chapter 6 - Pure Bending of Beams.pdfME 205- Chapter 6 - Pure Bending of Beams.pdf
ME 205- Chapter 6 - Pure Bending of Beams.pdf
 
办理哈珀亚当斯大学学院毕业证书文凭学位证书
办理哈珀亚当斯大学学院毕业证书文凭学位证书办理哈珀亚当斯大学学院毕业证书文凭学位证书
办理哈珀亚当斯大学学院毕业证书文凭学位证书
 
办理学位证(Massey证书)新西兰梅西大学毕业证成绩单原版一比一
办理学位证(Massey证书)新西兰梅西大学毕业证成绩单原版一比一办理学位证(Massey证书)新西兰梅西大学毕业证成绩单原版一比一
办理学位证(Massey证书)新西兰梅西大学毕业证成绩单原版一比一
 
定制英国克兰菲尔德大学毕业证成绩单原版一比一
定制英国克兰菲尔德大学毕业证成绩单原版一比一定制英国克兰菲尔德大学毕业证成绩单原版一比一
定制英国克兰菲尔德大学毕业证成绩单原版一比一
 
办理学位证(纽伦堡大学文凭证书)纽伦堡大学毕业证成绩单原版一模一样
办理学位证(纽伦堡大学文凭证书)纽伦堡大学毕业证成绩单原版一模一样办理学位证(纽伦堡大学文凭证书)纽伦堡大学毕业证成绩单原版一模一样
办理学位证(纽伦堡大学文凭证书)纽伦堡大学毕业证成绩单原版一模一样
 
原版定制卡尔加里大学毕业证(UC毕业证)留信学历认证
原版定制卡尔加里大学毕业证(UC毕业证)留信学历认证原版定制卡尔加里大学毕业证(UC毕业证)留信学历认证
原版定制卡尔加里大学毕业证(UC毕业证)留信学历认证
 
Ioannis Tzachristas Self-Presentation for MBA.pdf
Ioannis Tzachristas Self-Presentation for MBA.pdfIoannis Tzachristas Self-Presentation for MBA.pdf
Ioannis Tzachristas Self-Presentation for MBA.pdf
 
Storytelling, Ethics and Workflow in Documentary Photography
Storytelling, Ethics and Workflow in Documentary PhotographyStorytelling, Ethics and Workflow in Documentary Photography
Storytelling, Ethics and Workflow in Documentary Photography
 
Digital Marketing Training Institute in Mohali, India
Digital Marketing Training Institute in Mohali, IndiaDigital Marketing Training Institute in Mohali, India
Digital Marketing Training Institute in Mohali, India
 
Crack JAG. Guidance program for entry to JAG Dept. & SSB interview
Crack JAG. Guidance program for entry to JAG Dept. & SSB interviewCrack JAG. Guidance program for entry to JAG Dept. & SSB interview
Crack JAG. Guidance program for entry to JAG Dept. & SSB interview
 
Unlock Your Creative Potential: 7 Skills for Content Creator Evolution
Unlock Your Creative Potential: 7 Skills for Content Creator EvolutionUnlock Your Creative Potential: 7 Skills for Content Creator Evolution
Unlock Your Creative Potential: 7 Skills for Content Creator Evolution
 
定制(UQ毕业证书)澳洲昆士兰大学毕业证成绩单原版一比一
定制(UQ毕业证书)澳洲昆士兰大学毕业证成绩单原版一比一定制(UQ毕业证书)澳洲昆士兰大学毕业证成绩单原版一比一
定制(UQ毕业证书)澳洲昆士兰大学毕业证成绩单原版一比一
 
Black and White Minimalist Co Letter.pdf
Black and White Minimalist Co Letter.pdfBlack and White Minimalist Co Letter.pdf
Black and White Minimalist Co Letter.pdf
 
办理(Salford毕业证书)索尔福德大学毕业证成绩单原版一比一
办理(Salford毕业证书)索尔福德大学毕业证成绩单原版一比一办理(Salford毕业证书)索尔福德大学毕业证成绩单原版一比一
办理(Salford毕业证书)索尔福德大学毕业证成绩单原版一比一
 
Students with Oppositional Defiant Disorder
Students with Oppositional Defiant DisorderStudents with Oppositional Defiant Disorder
Students with Oppositional Defiant Disorder
 
LinkedIn for Your Job Search in April 2024
LinkedIn for Your Job Search in April 2024LinkedIn for Your Job Search in April 2024
LinkedIn for Your Job Search in April 2024
 

MCQs for Entrance Test for BN, MN, MSN Nursing by RS MEHTA

  • 1. MCQs FOR NURSES PREPARING FOR BN, B.SC.NURSING AND MN/M.SC. NURING ENTRANCE (GIFT FROM RAM SHARAN MEHTA) BPKIHS, DHARAN, NEPAL WISH YOU ALL THE BEST Maternal and Child Health Nursing 1. Accompanied by her husband, a patient seeks admission to the labor and delivery area. The client states that she is in labor, and says she attended the hospital clinic for prenatal care. Which question should the nurse ask her first? a. “Do you have any chronic illness?” b. “Do you have any allergies?” c. “What is your expected due date?” d. “Who will be with you during labor?” 2. A patient is in the second stage of labor. During this stage, how frequently should the nurse in charge assess her uterine contractions? a. Every 5 minutes b. Every 15 minutes c. Every 30 minutes d. Every 60 minutes 3. A patient is in last trimester of pregnancy. Nurse Jane should instruct her to notify her primary health care provider immediately if she notices: a. Blurred vision
  • 2. b. Hemorrhoids c. Increased vaginal mucus d. Shortness of breath on exertion 4. The nurse in charge is reviewing a patient’s prenatal history. Which finding indicates a genetic risk factor? a. The patient is 25 years old b. The patient has a child with cystic fibrosis c. The patient was exposed to rubella at 36 weeks’ gestation d. The patient has a history of preterm labor at 32 weeks’ gestation 5. A adult female patient is using the rhythm (calendar-basal body temperature) method of family planning. In this method, the unsafe period for sexual intercourse is indicated by; a. Return preovulatory basal body temperature b. Basal body temperature increase of 0.1 degrees to 0.2 degrees on the 2nd or 3rd day of cycle c. 3 full days of elevated basal body temperature and clear, thin cervical mucus d. Breast tenderness and mittelschmerz 6. During a nonstress test (NST), the electronic tracing displays a relatively flat line for fetal movement, making it difficult to evaluate the fetal heart rate (FHR). To mark the strip, the nurse in charge should instruct the client to push the control button at which time? a. At the beginning of each fetal movement b. At the beginning of each contraction c. After every three fetal movements d. At the end of fetal movement 7. When evaluating a client’s knowledge of symptoms to report during her pregnancy, which statement would indicate to the nurse in charge that the client understands the information given to her? a. “I’ll report increased frequency of urination.” b. “If I have blurred or double vision, I should call the clinic immediately.”
  • 3. c. “If I feel tired after resting, I should report it immediately.” d. “Nausea should be reported immediately.” 8. When assessing a client during her first prenatal visit, the nurse discovers that the client had a reduction mammoplasty. The mother indicates she wants to breast-feed. What information should the nurse give to this mother regarding breast-feeding success? a. “It’s contraindicated for you to breast-feed following this type of surgery.” b. “I support your commitment; however, you may have to supplement each feeding with formula.” c. “You should check with your surgeon to determine whether breast-feeding would be possible.” d. “You should be able to breast-feed without difficulty.” 9. Following a precipitous delivery, examination of the client’s vagina reveals a fourth-degree laceration. Which of the following would be contraindicated when caring for this client? a. Applying cold to limit edema during the first 12 to 24 hours b. Instructing the client to use two or more peripads to cushion the area c. Instructing the client on the use of sitz baths if ordered d. Instructing the client about the importance of perineal (Kegel) exercises 10. A client makes a routine visit to the prenatal clinic. Although she’s 14 weeks pregnant, the size of her uterus approximates that in an 18- to 20-week pregnancy. Dr. Diaz diagnoses gestational trophoblastic disease and orders ultrasonography. The nurse expects ultrasonography to reveal: a. an empty gestational sac. b. grapelike clusters. c. a severely malformed fetus. d. an extrauterine pregnancy. 11. After completing a second vaginal examination of a client in labor, the nurse-midwife determines that the fetus is in the right occiput anterior position and at –1 station. Based on these findings, the nurse-midwife knows that the fetal presenting part is:
  • 4. a. 1 cm below the ischial spines. b. directly in line with the ischial spines. c. 1 cm above the ischial spines. d. in no relationship to the ischial spines. 12. Which of the following would be inappropriate to assess in a mother who’s breast-feeding? a. The attachment of the baby to the breast. b. The mother’s comfort level with positioning the baby. c. Audible swallowing. d. The baby’s lips smacking 13. During a prenatal visit at 4 months gestation, a pregnant client asks whether tests can be done to identify fetal abnormalities. Between 18 and 40 weeks’ gestation, which procedure is used to detect fetal anomalies? a. Amniocentesis. b. Chorionic villi sampling. c. Fetoscopy. d. Ultrasound 14. A client, 30 weeks pregnant, is scheduled for a biophysical profile (BPP) to evaluate the health of her fetus. Her BPP score is 8. What does this score indicate? a. The fetus should be delivered within 24 hours. b. The client should repeat the test in 24 hours. c. The fetus isn’t in distress at this time. d. The client should repeat the test in 1 week. 15. A client who’s 36 weeks pregnant comes to the clinic for a prenatal checkup. To assess the client’s preparation for parenting, the nurse might ask which question? a. “Are you planning to have epidural anesthesia?”
  • 5. b. “Have you begun prenatal classes?” c. “What changes have you made at home to get ready for the baby?” d. “Can you tell me about the meals you typically eat each day?” 16. A client who’s admitted to labor and delivery has the following assessment findings: gravida 2 para 1, estimated 40 weeks’ gestation, contractions 2 minutes apart, lasting 45 seconds, vertex +4 station. Which of the following would be the priority at this time? a. Placing the client in bed to begin fetal monitoring. b. Preparing for immediate delivery. c. Checking for ruptured membranes. d. Providing comfort measures. 17. Nurse Roy is caring for a client in labor. The external fetal monitor shows a pattern of variable decelerations in fetal heart rate. What should the nurse do first? a. Change the client’s position. b. Prepare for emergency cesarean section. c. Check for placenta previa. d. Administer oxygen. 18. The nurse in charge is caring for a postpartum client who had a vaginal delivery with a midline episiotomy. Which nursing diagnosis takes priority for this client? a. Risk for deficient fluid volume related to hemorrhage b. Risk for infection related to the type of delivery c. Pain related to the type of incision d. Urinary retention related to periurethral edema 19. Which change would the nurse identify as a progressive physiological change in postpartum period? a. Lactation b. Lochia
  • 6. c. Uterine involution d. Diuresis 20. A 39-year-old at 37 weeks’ gestation is admitted to the hospital with complaints of vaginal bleeding following the use of cocaine 1 hour earlier. Which complication is most likely causing the client’s complaint of vaginal bleeding? a. Placenta previa b. Abruptio placentae c. Ectopic pregnancy d. Spontaneous abortion 21. A client with type 1 diabetes mellitus who’s a multigravida visits the clinic at 27 weeks gestation. The nurse should instruct the client that for most pregnant women with type 1 diabetes mellitus: a. Weekly fetal movement counts are made by the mother. b. Contraction stress testing is performed weekly. c. Induction of labor is begun at 34 weeks’ gestation. d. Nonstress testing is performed weekly until 32 weeks’ gestation 22. When administering magnesium sulfate to a client with preeclampsia, the nurse understands that this drug is given to: a. Prevent seizures b. Reduce blood pressure c. Slow the process of labor d. Increase dieresis 23. What’s the approximate time that the blastocyst spends traveling to the uterus for implantation? a. 2 days b. 7 days
  • 7. c. 10 days d. 14 weeks 24. After teaching a pregnant woman who is in labor about the purpose of the episiotomy, which of the following purposes stated by the client would indicate to the nurse that the teaching was effective? a. Shortens the second stage of labor b. Enlarges the pelvic inlet c. Prevents perineal edema d. Ensures quick placenta delivery 25. A primigravida client at about 35 weeks gestation in active labor has had no prenatal care and admits to cocaine use during the pregnancy. Which of the following persons must the nurse notify? a. Nursing unit manager so appropriate agencies can be notified b. Head of the hospital’s security department c. Chaplain in case the fetus dies in utero d. Physician who will attend the delivery of the infant 26. When preparing a teaching plan for a client who is to receive a rubella vaccine during the postpartum period, the nurse in charge should include which of the following? a. The vaccine prevents a future fetus from developing congenital anomalies b. Pregnancy should be avoided for 3 months after the immunization c. The client should avoid contact with children diagnosed with rubella d. The injection will provide immunity against the 7-day measles. 27. A client with eclampsia begins to experience a seizure. Which of the following would the nurse in charge do first? a. Pad the side rails b. Place a pillow under the left buttock
  • 8. c. Insert a padded tongue blade into the mouth d. Maintain a patent airway 28. While caring for a multigravida client in early labor in a birthing center, which of the following foods would be best if the client requests a snack? a. Yogurt b. Cereal with milk c. Vegetable soup d. Peanut butter cookies 29. The multigravida mother with a history of rapid labor who us in active labor calls out to the nurse, “The baby is coming!” which of the following would be the nurse’s first action? a. Inspect the perineum b. Time the contractions c. Auscultate the fetal heart rate d. Contact the birth attendant 30. While assessing a primipara during the immediate postpartum period, the nurse in charge plans to use both hands to assess the client’s fundus to: a. Prevent uterine inversion b. Promote uterine involution c. Hasten the puerperium period d. Determine the size of the fundus COMPREHENSIVES 1. Which individual is at greatest risk for developing hypertension? A) 45 year-old African American attorney B) 60 year-old Asian American shop owner
  • 9. C) 40 year-old Caucasian nurse D) 55 year-old Hispanic teacher 2. A child who ingested 15 maximum strength acetaminophen tablets 45 minutes ago is seen in the emergency department. Which of these orders should the nurse do first? A) Gastric lavage PRN B) Acetylcysteine (mucomyst) for age per pharmacy C) Start an IV Dextrose 5% with 0.33% normal saline to keep vein open D) Activated charcoal per pharmacy 3. Which complication of cardiac catheterization should the nurse monitor for in the initial 24 hours after the procedure? A) angina at rest B) thrombus formation C) dizziness D) falling blood pressure 4. A client is admitted to the emergency room with renal calculi and is complaining of moderate to severe flank pain and nausea. The client’s temperature is 100.8 degrees Fahrenheit. The priority nursing goal for this client is A) Maintain fluid and electrolyte balance B) Control nausea C) Manage pain D) Prevent urinary tract infection 5. What would the nurse expect to see while assessing the growth of children during their school age years? A) Decreasing amounts of body fat and muscle mass B) Little change in body appearance from year to year C) Progressive height increase of 4 inches each year D) Yearly weight gain of about 5.5 pounds per year
  • 10. 6. At a community health fair the blood pressure of a 62 year-old client is 160/96. The client states “My blood pressure is usually much lower.” The nurse should tell the client to A) go get a blood pressure check within the next 48 to 72 hours B) check blood pressure again in 2 months C) see the health care provider immediately D) visit the health care provider within 1 week for a BP check 7. The hospital has sounded the call for a disaster drill on the evening shift. Which of these clients would the nurse put first on the list to be discharged in order to make a room available for a new admission? A) A middle aged client with a history of being ventilator dependent for over 7 years and admitted with bacterial pneumonia five days ago B) A young adult with diabetes mellitus Type 2 for over 10 years and admitted with antibiotic induced diarrhea 24 hours ago C) An elderly client with a history of hypertension, hypercholesterolemia and lupus, and was admitted with Stevens-Johnson syndrome that morning D) An adolescent with a positive HIV test and admitted for acute cellulitus of the lower leg 48 hours ago 8. A client has been newly diagnosed with hypothyroidism and will take levothyroxine (Synthroid) 50 mcg/day by mouth. As part of the teaching plan, the nurse emphasizes that this medication: A) Should be taken in the morning B) May decrease the client’s energy level C) Must be stored in a dark container D) Will decrease the client’s heart rate 9. A 3 year-old child comes to the pediatric clinic after the sudden onset of findings that include irritability, thick muffled voice, croaking on inspiration, hot to touch, sit leaning forward, tongue protruding, drooling and suprasternal retractions. What should the nurse do first? A) Prepare the child for x-ray of upper airways B) Examine the child’s throat C) Collect a sputum specimen
  • 11. D) Notify the healthcare provider of the child’s status 10. In children suspected to have a diagnosis of diabetes, which one of the following complaints would be most likely to prompt parents to take their school age child for evaluation? A) Polyphagia B) Dehydration C) Bed wetting D) Weight loss 11. A client comes to the clinic for treatment of recurrent pelvic inflammatory disease. The nurse recognizes that this condition most frequently follows which type of infection? A) Trichomoniasis B) Chlamydia C) Staphylococcus D) Streptococcus 12. An RN who usually works in a spinal rehabilitation unit is floated to the emergency department. Which of these clients should the charge nurse assign to this RN? A) A middle-aged client who says "I took too many diet pills" and "my heart feels like it is racing out of my chest." B) A young adult who says "I hear songs from heaven. I need money for beer. I quit drinking 2 days ago for my family. Why are my arms and legs jerking?" C) An adolescent who has been on pain medications terminal cancer with an initial assessment finding pupils and a relaxed respiratory rate of 10 D) An elderly client who reports having taken a "large crack hit" 10 minutes prior to walking into the emergency room 13. When teaching a client with coronary artery disease about nutrition, the nurse should emphasize A) Eating 3 balanced meals a day B) Adding complex carbohydrates C) Avoiding very heavy meals
  • 12. D) Limiting sodium to 7 gms per day 14. Which of these findings indicate that a pump to deliver a basal rate of 10 ml per hour plus PRN for pain break through for morphine drip is not working? A) The client complains of discomfort at the IV insertion site B) The client states "I just can’t get relief from my pain." C) The level of drug is 100 ml at 8 AM and is 80 ml at noon D) The level of the drug is 100 ml at 8 AM and is 50 ml at noon 15. The nurse is speaking at a community meeting about personal responsibility for health promotion. A participant asks about chiropractic treatment for illnesses. What should be the focus of the nurse’s response? A) Electrical energy fields B) Spinal column manipulation C) Mind-body balance D) Exercise of joints 16. The nurse is performing a neurological assessment on a client post right CVA. Which finding, if observed by the nurse, would warrant immediate attention? A) Decrease in level of consciousness B) Loss of bladder control C) Altered sensation to stimuli D) Emotional ability 17. A child who has recently been diagnosed with cystic fibrosis is in a pediatric clinic where a nurse is performing an assessment. Which later finding of this disease would the nurse not expect to see at this time? A) Positive sweat test B) Bulky greasy stools C) Moist, productive cough D) Meconium ileus
  • 13. 18. The home health nurse visits a male client to provide wound care and finds the client lethargic and confused. His wife states he fell down the stairs 2 hours ago. The nurse should A) Place a call to the client’s health care provider for instructions B) Send him to the emergency room for evaluation C) Reassure the client’s wife that the symptoms are transient D) Instruct the client’s wife to call the doctor if his symptoms become worse 19. Which of the following should the nurse implement to prepare a client for a KUB (Kidney, Ureter, Bladder) radiograph test? A) Client must be NPO before the examination B) Enema to be administered prior to the examination C) Medicate client with Lasix 20 mg IV 30 minutes prior to the examination D) No special orders are necessary for this examination 20. The nurse is giving discharge teaching to a client 7 days post myocardial infarction. He asks the nurse why he must wait 6 weeks before having sexual intercourse. What is the best response by the nurse to this question? A) "You need to regain your strength before attempting such exertion." B) "When you can climb 2 flights of stairs without problems, it is generally safe.” C) "Have a glass of wine to relax you, then you can try to have sex." D) "If you can maintain an active walking program, you will have less risk." 21. A triage nurse has these 4 clients arrive in the emergency department within 15 minutes. Which client should the triage nurse send back to be seen first? A) A 2 month old infant with a history of rolling off the bed and has bulging fontanels with crying B) A teenager who got a singed beard while camping C) An elderly client with complaints of frequent liquid brown colored stools D) A middle aged client with intermittent pain behind the right scapula 22. While planning care for a toddler, the nurse teaches the parents about the expected developmental changes for this age. Which statement by the mother shows that she understands the child’s developmental needs?
  • 14. A) "I want to protect my child from any falls." B) "I will set limits on exploring the house." C) "I understand the need to use those new skills." D) "I intend to keep control over our child." 23. The nurse is preparing to administer an enteral feeding to a client via a nasogastric feeding tube. The most important action of the nurse is A) Verify correct placement of the tube B) Check that the feeding solution matches the dietary order C) Aspirate abdominal contents to determine the amount of last feeding remaining in stomach D) D) Ensure that feeding solution is at room temperature 24. The nurse is caring for a client with a serum potassium level of 3.5 mEq/L. The client is placed on a cardiac monitor and receives 40 mEq KCL in 1000 ml of 5% dextrose in water IV. Which of the following EKG patterns indicates to the nurse that the infusions should be discontinued? A) Narrowed QRS complex B) Shortened "PR" interval C) Tall peaked T waves D) Prominent "U" waves 25. A nurse prepares to care for a 4 year-old newly admitted for rhabdomyosarcoma. The nurse should alert the staff to pay more attention to the function of which area of the body? A) All striated muscles B) The cerebellum C) The kidneys D) The leg bones 26. The nurse anticipates that for a family who practices Chinese medicine the priority goal would be to A) Achieve harmony
  • 15. B) Maintain a balance of energy C) Respect life D) Restore yin and yang 27. During an assessment of a client with cardiomyopathy, the nurse finds that the systolic blood pressure has decreased from 145 to 110 mm Hg and the heart rate has risen from 72 to 96 beats per minute and the client complains of periodic dizzy spells. The nurse instructs the client to A) Increase fluids that are high in protein B) Restrict fluids C) Force fluids and reassess blood pressure D) D) Limit fluids to non-caffeine beverages 28. A client has a Swan-Ganz catheter in place. The nurse understands that this is intended to measure A) Right heart function B) Left heart function C) Renal tubule function D) Carotid artery function 29. A nurse enters a client’s room to discover that the client has no pulse or respirations. After calling for help, the first action the nurse should take is A) Start a peripheral IV B) Initiate closed-chest massage C) Establish an airway D) Obtain the crash cart 30. A client is receiving digoxin (Lanoxin) 0.25 mg. Daily. The health care provider has written a new order to give metoprolol (Lopressor) 25 mg. B.I.D. In assessing the client prior to administering the medications, which of the following should the nurse report immediately to the health care provider? A) Blood pressure 94/60 B) Heart rate 76
  • 16. C) Urine output 50 ml/hour D) Respiratory rate 16 31. While assessing a 1 month-old infant, which finding should the nurse report immediately? A) Abdominal respirations B) Irregular breathing rate C) Inspiratory grunt D) Increased heart rate with crying 32. The nurse practicing in a maternity setting recognizes that the post mature fetus is at risk due to A) Excessive fetal weight B) Low blood sugar levels C) Depletion of subcutaneous fat D) Progressive placental insufficiency 33. The nurse is caring for a client who had a total hip replacement 4 days ago. Which assessment requires the nurse’s immediate attention? A) I have bad muscle spasms in my lower leg of the affected extremity. B) "I just can’t ‘catch my breath’ over the past few minutes and I think I am in grave danger." C) "I have to use the bedpan to pass my water at least every 1 to 2 hours." D) "It seems that the pain medication is not working as well today." 34. A client has been taking furosemide (Lasix) for the past week. The nurse recognizes which finding may indicate the client is experiencing a negative side effect from the medication? A) Weight gain of 5 pounds B) Edema of the ankles C) Gastric irritability D) Decreased appetite
  • 17. 35. A client who is pregnant comes to the clinic for a first visit. The nurse gathers data about her obstetric history, which includes 3 year-old twins at home and a miscarriage 10 years ago at 12 weeks gestation. How would the nurse accurately document this information? A) Gravida 4 para 2 B) Gravida 2 para 1 C) Gravida 3 para 1 D) Gravida 3 para 2 36. The nurse is caring for a client with a venous stasis ulcer. Which nursing intervention would be most effective in promoting healing? A) Apply dressing using sterile technique B) Improve the client’s nutrition status C) Initiate limb compression therapy D) Begin proteolytic debridement 37. A nurse is to administer meperidine hydrochloride (Demerol) 100 mg, atropine sulfate (Atropisol) 0.4 mg, and promethizine hydrochloride (Phenergan) 50 mg IM to a pre-operative client. Which action should the nurse take first? A) Raise the side rails on the bed B) Place the call bell within reach C) Instruct the client to remain in bed D) D) Have the client empty bladder 38. Which of these statements best describes the characteristic of an effective reward-feedback system? A) Specific feedback is given as close to the event as possible B) Staff are given feedback in equal amounts over time C) Positive statements are to precede a negative statement D) Performance goals should be higher than what is attainable 39. A client with multiple sclerosis plans to begin an exercise program. In addition to discussing the benefits of regular exercise, the nurse should caution the client to avoid activities which
  • 18. A) Increase the heart rate B) Lead to dehydration C) Are considered aerobic D) May be competitive 40. During the evaluation of the quality of home care for a client with Alzheimer’s disease, the priority for the nurse is to reinforce which statement by a family member? A) At least 2 full meals a day is eaten. B) We go to a group discussion every week at our community center. C) We have safety bars installed in the bathroom and have 24 hour alarms on the doors. D) The medication is not a problem to have it taken 3 times a day. PHARMACOLOGY 1. The nursery nurse is putting erythromycin ointment in the newborn’s eyes to prevent infection. She places it in the following area of the eye: a. under the eyelid b. on the cornea. c. in the lower conjunctival sac d. by the optic disc. 2. The physician orders penicillin for a patient with streptococcal pharyngitis. The nurse administers the drug as ordered, and the patient has an allergic reaction. The nurse checks the medication order sheet and finds that the patient is allergic to penicillin. Legal responsibility for the error is: a. only the nurse’s—she should have checked the allergies before administering the medication. b. only the physician’s—she gave the order, the nurse is obligated to follow it. c. only the pharmacist’s—he should alert the floor to possible allergic reactions. d. the pharmacist, physician, and nurse are all liable for the mistake 3. James Perez, a nurse on a geriatric floor, is administering a dose of digoxin to one of his patients. The woman asks why she takes a different pill than her niece, who also has heart trouble. James replies that as people get older, liver and kidney function decline, and if the dose is as high as her niece’s, the drug will tend to: a. have a shorter half-life. b. accumulate. c. have decreased distribution. d. have increased absorption.
  • 19. 4. The nurse is administering Augmentin to her patient with a sinus infection. Which is the best way for her to insure that she is giving it to the right patient? a. Call the patient by name b. Read the name of the patient on the patient’s door c. Check the patient’s wristband d. Check the patient’s room number on the unit census list 5. The most important instructions a nurse can give a patient regarding the use of the antibiotic Ampicillin prescribed for her are to a. call the physician if she has any breathing difficulties. b. take it with meals so it doesn’t cause an upset stomach. c. take all of the medication prescribed even if the symptoms stop sooner. d. not share the pills with anyone else. 6. Mr. Jessie Ray, a newly admitted patient, has a seizure disorder which is being treated with medication. Which of the following drugs would the nurse question if ordered for him? a. Phenobarbitol, 150 mg hs b. Amitriptylene (Elavil), 10 mg QID. c. Valproic acid (Depakote), 150 mg BID d. Phenytoin (Dilantin), 100 mg TID 7. Mrs. Jane Gately has been dealing with uterine cancer for several months. Pain management is the primary focus of her current admission to your oncology unit. Her vital signs on admission are BP 110/64, pulse 78, respirations 18, and temperature 99.2 F. Morphine sulfate 6mg IV, q 4 hours, prn has been ordered. During your assessment after lunch, your findings are: BP 92/60, pulse 66, respirations 10, and temperature 98.8. Mrs. Gately is crying and tells you she is still experiencing severe pain. Your action should be to a. give her the next ordered dose of MS. b. give her a back rub, put on some light music, and dim the lights in the room. c. report your findings to the RN, requesting an alternate medication order d. be obtained from the physician. e. call her daughter to come and sit with her. 8. When counseling a patient who is starting to take MAO (monoamine oxidase) inhibitors such as Nardil for depression, it is essential that they be warned not to eat foods containing tyramine, such as: a. Roquefort, cheddar, or Camembert cheese. b. grape juice, orange juice, or raisins. c. onions, garlic, or scallions. d. ground beef, turkey, or pork. 9. The physician orders an intramuscular injection of Demerol for the postoperativepatient’s pain. When preparing to draw up the medication, the nurse is careful to remove the correct vial from the narcotics cabinet. It is labeled a. simethicone. b. albuterol. c. meperidine. d. ibuprofen.
  • 20. 10. The nurse is administering an antibiotic to her pediatric patient. She checks the patient’s armband and verifies the correct medication by checking the physician’s order, medication kardex, and vial. Which of the following is not considered one of the five “rights” of drug administration? a. Right dose b. Right route c. Right frequency d. Right time 11. A nurse is preparing the client’s morning NPH insulin dose and notices a clumpy precipitate inside the insulin vial. The nurse should: a. draw up and administer the dose b. shake the vial in an attempt to disperse the clumps c. draw the dose from a new vial d. warm the bottle under running water to dissolve the clump 12. A client with histoplasmosis has an order for ketoconazole (Nizoral). The nurse teaches the client to do which of the following while taking this medication? a. take the medication on an empty stomach b. b. take the medication with an antacid c. c. avoid exposure to sunlight d. d. limit alcohol to 2 ounces per day 13. A nurse has taught a client taking a xanthine bronchodilator about beverages to avoid. The nurse determines that the client understands the information if the client chooses which of the following beverages from the dietary menu? a. chocolate milk b. cranberry juice c. coffee d. cola 14. A client is taking famotidine (Pepcid) asks the home care nurse what would be the best medication to take for a headache. The nurse tells the client that it would be best to take: a. aspirin (acetylsalicylic acid, ASA) b. b. ibuprofen (Motrin) c. c. acetaminophen (Tylenol) d. d. naproxen (Naprosyn) 15. A nurse is planning dietary counseling for the client taking triamterene (Dyrenium). The nurse plans to include which of the following in a list of foods that are acceptable? a. baked potato b. b. bananas c. c. oranges d. d. pears canned in water 16. A client with advanced cirrhosis of the liver is not tolerating protein well, as eveidenced by abnormal laboratory values. The nurse anticipates that which of the following medications will be prescribed for the client? a. lactulose (Chronulac) b. ethacrynic acid (Edecrin) c. folic acid (Folvite) d. thiamine (Vitamin B1)
  • 21. 17. A female client tells the clinic nurse that her skin is very dry and irritated. Which product would the nurse suggest that the client apply to the dry skin? a. glycerin emollient b. aspercreame c. myoflex d. acetic acid solution 18. A nurse is providing instructions to a client regarding quinapril hydrochloride (Accupril). The nurse tells the client: a. to take the medication with food only b. to rise slowly from a lying to a sitting position c. to discontinue the medication if nausea occurs d. that a therapeutic effect will be noted immediately 19. Auranofin (Ridaura) is prescribed for a client with rheumatoid arthritis, and the nurse monitors the client for signs of an adverse effect related to the medication. Which of the following indicates an adverse effect? a. nausea b. b. diarrhea c. c. anorexia d. d. proteinuria 20. A client has been taking benzonatate (Tessalon) as ordered. The nurse tells the client that this medication should do which of the following? a. take away nausea and vomiting b. calm the persistent cough c. decrease anxiety level d. increase comfort level Answers: 1. C. The ointment is placed in the lower conjunctival sac so it will not scratch the eye itself and will get well distributed. 2. D. The physician, nurse, and pharmacist all are licensed professionals and share responsibility for errors. 3. B. The decreased circulation to the kidney and reduced liver function tend to allow drugs to accumulate and have toxic effects. 4. C. The correct way to identify a patient before giving a medication is to check the name on the medication administration record with the patient’s identification band. The nurse should also ask the patient to state their name. The name on the door or the census list are not sufficient proof of identification. Calling the patient by name is not as effective as having the patient state their name; patients may not hear well or understand what the nurse is saying, and may respond to a name which is not their own.
  • 22. 5. C. Frequently patients do not complete an entire course of antibiotic therapy, and the bacteria are not destroyed. 6. B. Elavil is an antidepressant that lowers the seizure threshold, so would not be appropriate for this patient. The other medications are anti-seizure drugs. 7. C. Morphine sulfate depresses the respiratory center. When the rate is less than 10, the MD should be notified. 8. A. Monoamine oxidase inhibitors react with foods high in the amino acid tyramine to cause dangerously high blood pressure. Aged cheeses are all high in this amino acid; the other foods are not. 9. C. The generic name for Demerol is meperidine. 10. C. The five rights of medication administration are right drug, right dose, right route, right time, right patient. Frequency is not included. 11. C. The nurse should always inspect the vial of insulin before use for solution changes that may signify loss of potency. NPH insulin is normally uniformly cloudy. Clumping, frosting, and precipitates are signs of insulin damage. In this situation, because potency is questionable, it is safer to discard the vial and draw up the dose from a new vial. 12. C. The client should be taught that ketoconazole is an antifungal medication. It should be taken with food or milk. Antacids should be avoided for 2 hours after it is taken because gastric acid is needed to activate the medication. The client should avoid concurrent use of alcohol, because the medication is hepatotoxic. The client should also avoid exposure to sunlight, because the medication increases photosensitivity. 13. B. Cola, coffee, and chocolate contain xanthine and should be avoided by the client taking a xanthine bronchodilator. This could lead to an increased incidence of cardiovascular and central nervous system side effects that can occur with the use of these types of bronchodilators. 14. C. The client is taking famotidine, a histamine receptor antagonist. This implies that the client has a disorder characterized by gastrointestinal (GI) irritation. The only medication of the ones listed in the options that is not irritating to the GI tract is acetaminophen. The other medications could aggravate an already existing GI problem. 15. D. Triamterene is a potassium-sparing diuretic, and clients taking this medication should be cautioned against eating foods that are high in potassium, including many vegetables, fruits, and fresh meats. Because potassium is very water-soluble, foods that are prepared in water are often lower in potassium. 16. A. The client with cirrhosis has impaired ability to metabolize protein because of liver dysfunction. Administration of lactulose aids in the clearance of ammonia via the
  • 23. gastrointestinal (GI) tract. Ethacrynic acid is a diuretic. Folic acid and thiamine are vitamins, which may be used in clients with liver disease as supplemental therapy. 17. A. Glycerin is an emollient that is used for dry, cracked, and irritated skin. Aspercreame and Myoflex are used to treat muscular aches. Acetic acid solution is used for irrigating, cleansing, and packing wounds infected by Pseudomonas aeruginosa. 18. B. Accupril is an angiotensin-converting enzyme (ACE) inhibitor. It is used in the treatment of hypertension. The client should be instructed to rise slowly from a lying to sitting position and to permit the legs to dangle from the bed momentarily before standing to reduce the hypotensive effect. The medication does not need to be taken with meals. It may be given without regard to food. If nausea occurs, the client should be instructed to take a noncola carbonated beverage and salted crackers or dry toast. A full therapeutic effect may be noted in 1 to 2 weeks. 19. D. Auranofin (Ridaura) is a gold preparation that is used as an antirheumatic. Gold toxicity is an adverse effect and is evidenced by decreased hemoglobin, leukopenia, reduced granulocyte counts, proteinuria, hematuria, stomatitis, glomerulonephritis, nephrotic syndrome, or cholestatic jaundice. Anorexia, nausea, and diarrhea are frequent side effects of the medication. 20. B. Benzonatate is a locally acting antitussive. Its effectiveness is measured by the degree to which it decreases the intensity and frequency of cough, without eliminating the cough reflex. PEDIATRIC 1. The parents of a child, age 5, who will begin school in the fall ask the nurse for anticipatory guidance. The nurse should explain that a child of this age: a. Still depends on the parents b. Rebels against scheduled activities c. Is highly sensitive to criticism d. Loves to tattle 2. While preparing to discharge an 8-month-old infant who is recovering from gastroenteritis and dehydration, the nurse teaches the parents about their infant’s dietary and fluid requirements. The nurse should include which other topic in the teaching session? a. Nursery schools b. Toilet Training c. Safety guidelines d. Preparation for surgery 3. Nurse Betina should begin screening for lead poisoning when a child reaches which age? a. 6 months b. 12 months
  • 24. c. 18 months d. 24 months 4. When caring for an 11-month-old infant with dehydration and metabolic acidosis, the nurse expects to see which of the following? a. A reduced white blood cell count b. A decreased platelet count c. Shallow respirations d. Tachypnea 5. After the nurse provides dietary restrictions to the parents of a child with celiac disease, which statement by the parents indicates effective teaching? a. “Well follow these instructions until our child’s symptoms disappear.” b. “Our child must maintain these dietary restrictions until adulthood.” c. “Our child must maintain these dietary restrictions lifelong.” d. “We’ll follow these instructions until our child has completely grown and developed.” 6. A parent brings a toddler, age 19 months, to the clinic for a regular check-up. When palpating the toddler’s fontanels, what should the nurse expects to find? a. Closed anterior fontanel and open posterior fontanel b. Open anterior and fontanel and closed posterior fontanel c. Closed anterior and posterior fontanels d. Open anterior and posterior fontanels 7. Patrick, a healthy adolescent has meningitis and is receiving I.V. and oral fluids. The nurse should monitor this client’s fluid intake because fluid overload may cause: a. Cerebral edema b. Dehydration c. Heart failure d. Hypovolemic shock 8. An infant is hospitalized for treatment of nonorganic failure to thrive. Which nursing action is most appropriate for this infant? a. Encouraging the infant to hold a bottle b. Keeping the infant on bed rest to conserve energy c. Rotating caregivers to provide more stimulation d. Maintaining a consistent, structured environment 9. The mother of Gian, a preschooler with spina bifida tells the nurse that her daughter sneezes and gets a rash when playing with brightly colored balloons, and that she recently had an allergic reaction after eating kiwifruit and bananas. The nurse would suspect that the child may have an allergy to: a. Bananas b. Latex c. Kiwifruit d. Color dyes
  • 25. 10. Cristina, a mother of a 4-year-old child tells the nurse that her child is a very poor eater. What’s the nurse’s best recommendation for helping the mother increase her child’s nutritional intake? a. Allow the child to feed herself b. Use specially designed dishes for children – for example, a plate with the child’s favorite cartoon character c. Only serve the child’s favorite foods d. Allow the child to eat at a small table and chair by herself 11. Nurse Roy is administering total parental nutrition (TPN) through a peripheral I.V. line to a school-age child. What’s the smallest amount of glucose that’s considered safe and not caustic to small veins, while also providing adequate TPN? a. 5% glucose b. 10% glucose c. 15% glucose d. 17% glucose 12. David, age 15 months, is recovering from surgery to remove Wilms’ tumor. Which findings best indicates that the child is free from pain? a. Decreased appetite b. Increased heart rate c. Decreased urine output d. Increased interest in play 13. When planning care for a 8-year-old boy with Down syndrome, the nurse should: a. Plan interventions according to the developmental level of a 7-year-old child because that’s the child’s age b. Plan interventions according to the developmental levels of a 5-year-old because the child will have developmental delays c. Assess the child’s current developmental level and plan care accordingly d. Direct all teaching to the parents because the child can’t understand 14. Nurse Victoria is teaching the parents of a school-age child. Which teaching topic should take priority? a. Prevent accidents b. Keeping a night light on to allay fears c. Explaining normalcy of fears about body integrity d. Encouraging the child to dress without help 15. The nurse is finishing her shift on the pediatric unit. Because her shift is ending, which intervention takes top priority? a. Changing the linens on the clients’ beds b. Restocking the bedside supplies needed for a dressing change on the upcoming shift c. Documenting the care provided during her shift d. Emptying the trash cans in the assigned client room
  • 26. 16. Nurse Alice is providing cardiopulmonary resuscitation (CPR) to a child, age 4. the nurse should: a. Compress the sternum with both hands at a depth of 1½ to 2” (4 to 5 cm) b. Deliver 12 breaths/minute c. Perform only two-person CPR d. Use the heel of one hand for sternal compressions 17. A 4-month-old with meningococcal meningitis has just been admitted to the pediatric unit. Which nursing intervention has the highest priority? a. Instituting droplet precautions b. Administering acetaminophen (Tylenol) c. Obtaining history information from the parents d. Orienting the parents to the pediatric unit 18. Sheena, tells the nurse that she wants to begin toilet training her 22-month-old child. The most important factor for the nurse to stress to the mother is: a. Developmental readiness of the child b. Consistency in approach c. The mother’s positive attitude d. Developmental level of the child’s peers 19. An infant who has been in foster care since birth requires a blood transfusion. Who is authorized to give written, informed consent for the procedure? a. The foster mother b. The social worker who placed the infant in the foster home c. The registered nurse caring for the infant d. The nurse-manager 20. A child is undergoing remission induction therapy to treat leukemia. Allopurinol is included in the regimen. The main reason for administering allopurinol as part of the client’s chemotherapy regimen is to: a. Prevent metabolic breakdown of xanthine to uric acid b. Prevent uric acid from precipitating in the ureters c. Enhance the production of uric acid to ensure adequate excretion of urine d. Ensure that the chemotherapy doesn’t adversely affect the bone marrow 21. A 10-year-old client contracted severe acute respiratory syndrome (SARS) when traveling abroad with her parents. The nurse knows she must put on personal protective equipment to protect herself while providing care. Based on the mode of SARS transmission, which personal protective should the nurse wear? a. Gloves b. Gown and gloves c. Gown, gloves, and mask d. Gown, gloves, mask, and eye goggles or eye shield
  • 27. 22. A tuberculosis intradermal skin test to detect tuberculosis infection is given to a high-risk adolescent. How long after the test is administered should the result be evaluated? a. Immediately b. Within 24 hours c. In 48 to 72 hours d. After 5 days 23. Nurse Oliver s teaching a mother who plans to discontinue breast-feeding after 5 months. The nurse should advise her to include which foods in her infant’s diet? a. Iron-rich formula and baby food b. Whole milk and baby food c. Skim milk and baby food d. Iron-rich formula only 24. Gracie, the mother of a 3-month-old infant calls the clinic and states that her child has a diaper rash. What should the nurse advise? a. “Switch to cloth diapers until the rash is gone” b. “Use baby wipes with each diaper change.” c. “Leave the diaper off while the infant sleeps.” d. “Offer extra fluids to the infant until the rash improves.” 25. Nurse Kelly is teaching the parents of a young child how to handle poisoning. If the child ingests poison, what should the parents do first? a. Administer ipecac syrup b. Call an ambulance immediately c. Call the poison control center d. Punish the child for being bad 26. A child has third-degree burns of the hands, face, and chest. Which nursing diagnosis takes priority? a. Ineffective airway clearance related to edema b. Disturbed body image related to physical appearance c. Impaired urinary elimination related to fluid loss d. Risk for infection related to epidermal disruption 27. A 3-year-old child is receiving dextrose 5% in water and half-normal saline solution at 100 ml/hour. Which sign or symptom suggests excessive I.V. fluid intake? a. Worsening dyspnea b. Gastric distension c. Nausea and vomiting d. Temperature of 102°F (38.9° C) 28. Which finding would alert a nurse that a hospitalized 6-year-old child is at risk for a severe asthma exacerbation? a. Oxygen saturation of 95% b. Mild work of breathing
  • 28. c. Absence of intercostals or substernal retractions d. History of steroid-dependent asthma 29. Nurse Mariane is caring for an infant with spina bifida. Which technique is most important in recognizing possible hydrocephalus? a. Measuring head circumference b. Obtaining skull X-ray c. Performing a lumbar puncture d. Magnetic resonance imaging (MRI) 30. An adolescent who sustained a tibia fracture in a motor vehicle accident has a cast. What should the nurse do to help relieve the itching? a. Apply cool air under the cast with a blow-dryer b. Use sterile applicators to scratch the itch c. Apply cool water under the cast d. Apply hydrocortisone cream under the cast using sterile applicator. Questions: 1. Which of the following would be inappropriate when administering chemotherapy to a child? a. Monitoring the child for both general and specific adverse effects b. Observing the child for 10 minutes to note for signs of anaphylaxis c. Administering medication through a free-flowing intravenous line d. Assessing for signs of infusion infiltration and irritation 2. Which of the following is the best method for performing a physical examination on a toddler a. From head to toe b. Distally to proximally c. From abdomen to toes, the to head d. From least to most intrusive 3. Which of the following organisms is responsible for the development of rheumatic fever? a. Streptococcal pneumonia b. Haemophilus influenza c. Group A β-hemolytic streptococcus
  • 29. d. Staphylococcus aureus 4. Which of the following is most likely associated with a cerebrovascular accident (CVA) resulting from congenital heart disease? a. Polycythemia b. Cardiomyopathy c. Endocarditis d. Low blood pressure 5. How does the nurse appropriately administer mycostatin suspension in an infant? a. Have the infant drink water, and then administer mycostatin in a syringe b. Place mycostatin on the nipple of the feeding bottle and have the infant suck it c. Mix mycostatin with formula d. Swab mycostatin on the affected areas 6. A mother tells the nurse that she is very worried because her 2-year old child does not finish his meals. What should the nurse advise the mother? a. make the child seat with the family in the dining room until he finishes his meal b. provide quiet environment for the child before meals c. do not give snacks to the child before meals d. put the child on a chair and feed him 7. The nurse is assessing a newborn who had undergone vaginal delivery. Which of the following findings is least likely to be observed in a normal newborn? a. uneven head shape b. respirations are irregular, abdominal, 30-60 bpm c. (+) moro reflex d. heart rate is 80 bpm 8. Which of the following situations increase risk of lead poisoning in children?
  • 30. a. playing in the park with heavy traffic and with many vehicles passing by b. playing sand in the park c. playing plastic balls with other children d. playing with stuffed toys at home 9. An inborn error of metabolism that causes premature destruction of RBC? a. G6PD b. Hemocystinuria c. Phenylketonuria d. Celiac Disease 10. Which of the following blood study results would the nurse expect as most likely when caring for the child with iron deficiency anemia? a. Increased hemoglobin b. Normal hematocrit c. Decreased mean corpuscular volume (MCV) d. Normal total iron-binding capacity (TIBC) 11. The nurse answers a call bell and finds a frightened mother whose child, the patient, is having a seizure. Which of these actions should the nurse take? a. The nurse should insert a padded tongue blade in the patient’s mouth to prevent the child from swallowing or choking on his tongue. b. The nurse should help the mother restrain the child to prevent him from injuring himself. c. The nurse should call the operator to page for seizure assistance. d. The nurse should clear the area and position the client safely. 12. At the community center, the nurse leads an adolescent health information group, which often expands into other areas of discussion. She knows that these youths are trying to find out “who they are,” and discussion often focuses on which directions they want to take in school and life, as well as peer relationships. According to Erikson, this stage is known as:
  • 31. a. identity vs. role confusion. b. adolescent rebellion. c. career experimentation. d. relationship testing 13. The nurse is assessing a 9-month-old boy for a well-baby check up. Which of the following observations would be of most concern? a. The baby cannot say “mama” when he wants his mother. b. The mother has not given him finger foods. c. The child does not sit unsupported. d. The baby cries whenever the mother goes out. 14. Cheska, the mother of an 11-month-old girl, KC, is in the clinic for her daughter’s immunizations. She expresses concern to the nurse that Shannon cannot yet walk. The nurse correctly replies that, according to the Denver Developmental Screen, the median age for walking is: a. 12 months. b. 15 months. c. 10 months. d. 14 months. 15. Sally Kent., age 13, has had a lumbar puncture to examine the CSF to determine if bacterial infection exists. The best position to keep her in after the procedure is: a. prone for two hours to prevent aspiration, should she vomit. b. semi-fowler’s so she can watch TV for five hours and be entertained. c. supine for several hours, to prevent headache. d. on her right sides to encourage return of CSF 16. Buck’s traction with a 10 lb. weight is securing a patient’s leg while she is waiting for surgery to repair a hip fracture. It is important to check circulation- sensation-movement:
  • 32. a. every shift. b. every day. c. every 4 hours. d. every 15 minutes. 17. Carol Smith is using bronchodilators for asthma. The side effects of these drugs that you need to monitor this patient for include: a. tachycardia, nausea, vomiting, heart palpitations, inability to sleep, restlessness, and seizures. b. tachycardia, headache, dyspnea, temp . 101 F, and wheezing. c. blurred vision, tachycardia, hypertension, headache, insomnia, and oliguria. d. restlessness, insomnia, blurred vision, hypertension, chest pain, and muscle weakness. 18. The adolescent patient has symptoms of meningitis: nuchal rigidity, fever, vomiting, and lethargy. The nurse knows to prepare for the following test: a. blood culture. b. throat and ear culture. c. CAT scan. d. lumbar puncture. 19. The nurse is drawing blood from the diabetic patient for a glycosolated hemoglobin test. She explains to the woman that the test is used to determine: a. the highest glucose level in the past week. b. her insulin level. c. glucose levels over the past several months. d. her usual fasting glucose level. 20. The twelve-year-old boy has fractured his arm because of a fall from his bike. After the injury has been casted, the nurse knows it is most important to perform all of the following assessments on the area distal to the injury except:
  • 33. a. capillary refill. b. radial and ulnar pulse. c. finger movement d. skin integrity Answers: 1. B. When administering chemotherapy, the nurse should observe for an anaphylactic reaction for 20 minutes and stop the medication if one is suspected. Chemotherapy is associated with both general and specific adverse effects, therefore close monitoring for them is important. 2. D. When examining a toddler or any small child, the best way to perform the exam is from least to most intrusive. Starting at the head or abdomen is intrusive and should be avoided. Proceeding from distal to proximal is inappropriate at any age. 3. C. Rheumatic fever results as a delayed reaction to inadequately treated group A β-hemolytic streptococcal infection. 4. A. The child with congenital heart disease develops polycythemia resulting from an inadequate mechanism to compensate for decreased oxygen saturation 5. D. Mycostatin suspension is given as swab. Never mix medications with food and formula. 6. C. If the child is hungry he/she more likely would finish his meals. Therefore, the mother should be advised not to give snacks to the child. The child is a “busy toddler.” He/she will not able to keep still for a long time. 7. D. Normal heart rate of the newborn is 120 to 160 bpm. Choices A, B, and C are normal assessment findings (uneven head shape is molding). 8. A. Lead poisoning may be caused by inhalation of dusk and smoke from leaded gas. It may also be caused by lead-based paint, soil, water (especially from plumbings of old houses). 9. A. Glucose-6-phosphate dehydrogenase deficiency (G6PD) is an X-linked recessive hereditary disease characterised by abnormally low levels of glucose-6-phosphate dehydrogenase (abbreviated G6PD or G6PDH), a metabolic enzyme involved in the pentose phosphate pathway, especially important in red blood cell metabolism.
  • 34. 10. C. For the child with iron deficiency anemia, the blood study results most likely would reveal decreased mean corpuscular volume (MCV) which demonstrates microcytic anemia, decreased hemoglobin, decreased hematocrit and elevated total iron binding capacity. 11. D. The primary role of the nurse when a patient has a seizure is to protect the patient from harming him or herself. 12. A. During this period, which lasts up to the age of 18-21 years, the individual develops a sense of “self.” Peers have a major big influence over behavior, and the major decision is to determine a vocational goal. 13. C. Over 90% percent of babies can sit unsupported by nine months. Most babies cannot say “mama” in the sense that it refers to their mother at this time. 14. A. By 12 months, 50 percent of children can walk well. 15. C. Lying flat keeps the patient from having a “spinal headache.” Increasing the fluid intake will assist in replenishing the lost fluid during this time. 16. C. The patient can lose vascular status without the nurse being aware if left for more than 4 hours, yet checks should not be so frequent that the patient becomes anxious. Vital signs are generally checked q4h, at which time the CSM checks can easily be performed. 17. A. Bronchodilators can produce the side effects listed in answer choice (A) for a short time after the patient begins using them. 18. D. Meningitis is an infection of the meninges, the outer membrane of the brain. Since it is surrounded by cerebrospinal fluid, a lumbar puncture will help to identify the organism involved. 19. C. The glycosolated hemoglobin test measures glucose levels for the previous 3 to 4 months. 20. D. Capillary refill, pulses, and skin temperature and color are indicative of intact circulation and absence of compartment syndrome. Skin integrity is less important. 50 ITEM MEDICAL SURGICAL 1. The nurse is performing her admission assessment of a patient. When grading arterial pulses, a 1+ pulse indicates: a. Above normal perfusion. b. Absent perfusion. c. Normal perfusion. d. Diminished perfusion. 2. Murmurs that indicate heart disease are often accompanied by other symptoms such as: a. Dyspnea on exertion. b. Subcutaneous emphysema. c. Thoracic petechiae.
  • 35. d. Periorbital edema. 3. Which pregnancy-related physiologic change would place the patient with a history of cardiac disease at the greatest risk of developing severe cardiac problems? a. Decrease heart rate b. Decreased cardiac output c. Increased plasma volume d. Increased blood pressure 4. The priority nursing diagnosis for the patient with cardiomyopathy is: a. Anxiety related to risk of declining health status. b. Ineffective individual coping related to fear of debilitating illness c. Fluid volume excess related to altered compensatory mechanisms. d. Decreased cardiac output related to reduced myocardial contractility. 5. A patient with thrombophlebitis reached her expected outcomes of care. Her affected leg appears pink and warm. Her pedal pulse is palpable and there is no edema present. Which step in the nursing process is described above? a. Planning b. Implementation c. Analysis d. Evaluation 6. An elderly patient may have sustained a basilar skull fracture after slipping and falling on an icy sidewalk. The nurse knows that basilar skull factures: a. Are the least significant type of skull fracture. b. May have cause cerebrospinal fluid (CSF) leaks from the nose or ears. c. Have no characteristic findings. d. Are always surgically repaired. 7. Which of the following types of drugs might be given to control increased intracranial pressure (ICP)? a. Barbiturates b. Carbonic anhydrase inhibitors c. Anticholinergics d. Histamine receptor blockers 8. The nurse is teaching family members of a patient with a concussion about the early signs of increased intracranial pressure (ICP). Which of the following would she cite as an early sign of increased ICP? a. Decreased systolic blood pressure b. Headache and vomiting c. Inability to wake the patient with noxious stimuli d. Dilated pupils that don’t react to light 9. Jessie James is diagnosed with retinal detachment. Which intervention is the most important for this patient? a. Admitting him to the hospital on strict bed rest b. Patching both of his eyes c. Referring him to an ophthalmologist d. Preparing him for surgery 10. Dr. Bruce Owen, a chemist, sustained a chemical burn to one eye. Which intervention takes priority for a patient with a chemical burn of the eye?
  • 36. a. Patch the affected eye and call the ophthalmologist. b. Administer a cycloplegic agent to reduce ciliary spasm. c. Immediately instill a tropical anesthetic, then irrigate the eye with saline solution. d. Administer antibiotics to reduce the risk of infection 11. The nurse is assessing a patient and notes a Brudzinski’s sign and Kernig’s sign. These are two classic signs of which of the following disorders? a. Cerebrovascular accident (CVA) b. Meningitis c. Seizure disorder d. Parkinson’s disease 12. A patient is admitted to the hospital for a brain biopsy. The nurse knows that the most common type of primary brain tumor is: a. Meningioma. b. Angioma. c. Hemangioblastoma. d. Glioma. 13. The nurse should instruct the patient with Parkinson’s disease to avoid which of the following? a. Walking in an indoor shopping mall b. Sitting on the deck on a cool summer evening c. Walking to the car on a cold winter day d. Sitting on the beach in the sun on a summer day 14. Gary Jordan suffered a cerebrovascular accident that left her unable to comprehend speech and unable to speak. This type of aphasia is known as: a. Receptive aphasia b. Expressive aphasia c. Global aphasia d. Conduction aphasia 15. Kelly Smith complains that her headaches are occurring more frequently despite medications. Patients with a history of headaches should be taught to avoid: a. Freshly prepared meats. b. Citrus fruits. c. Skim milk d. Chocolate 16. Immediately following cerebral aneurysm rupture, the patient usually complains of: a. Photophobia b. Explosive headache c. Seizures d. Hemiparesis 17. Which of the following is a cause of embolic brain injury? a. Persistent hypertension b. Subarachnoid hemorrhage c. Atrial fibrillation d. Skull fracture 18. Although Ms. Priestly has a spinal cord injury, she can still have sexual intercourse. Discharge teaching should make her aware that:
  • 37. a. She must remove indwelling urinary catheter prior to intercourse. b. She can no longer achieve orgasm. c. Positioning may be awkward. d. She can still get pregnant. 19. Ivy Hopkins, age 25, suffered a cervical fracture requiring immobilization with halo traction. When caring for the patient in halo traction, the nurse must: a. Keep a wrench taped to the halo vest for quick removal if cardiopulmonary resuscitation is necessary. b. Remove the brace once a day to allow the patient to rest. c. Encourage the patient to use a pillow under the ring. d. Remove the brace so that the patient can shower. 20. The nurse asks a patient’s husband if he understands why his wife is receiving nimodipine (Nimotop), since she suffered a cerebral aneurysm rupture. Which response by the husband indicates that he understands the drug’s use? a. “Nimodipine replaces calcium.” b. “Nimodipine promotes growth of blood vessels in the brain.” c. “Nimodipine reduces the brain’s demand for oxygen.” d. “Nimodipine reduces vasospasm in the brain.” 21. Many men who suffer spinal injuries continue to be sexually active. The teaching plan for a man with a spinal cord injury should include sexually concerns. Which of the following injuries would most likely prevent erection and ejaculation? a. C5 b. C7 c. T4 d. S4 22. Cathy Bates, age 36, is a homemaker who frequently forgets to take her carbamazepine (Tegretol). As a result, she has been experiencing seizures. How can the nurse best help the patient remember to take her medication? a. Tell her take her medication at bedtime. b. Instruct her to take her medication after one of her favorite television shows. c. Explain that she should take her medication with breakfast. d. Tell her to buy an alarm watch to remind her. 23. Richard Barnes was diagnosed with pneumococcal meningitis. What response by the patient indicates that he understands the precautions necessary with this diagnosis? a. “I’m so depressed because I can’t have any visitors for a week.” b. “Thank goodness, I’ll only be in isolation for 24 hours.” c. “The nurse told me that my urine and stool are also sources of meningitis bacteria.” d. “The doctor is a good friend of mine and won’t keep me in isolation.” 24. An early symptom associated with amyotrophic lateral sclerosis (ALS) includes: a. Fatigue while talking b. Change in mental status c. Numbness of the hands and feet d. Spontaneous fractures 25. When caring for a patient with esophageal varices, the nurse knows that bleeding in this disorder usually stems from:
  • 38. a. Esophageal perforation b. Pulmonary hypertension c. Portal hypertension d. Peptic ulcers 26. Tiffany Black is diagnosed with type A hepatitis. What special precautions should the nurse take when caring for this patient? a. Put on a mask and gown before entering the patient’s room. b. Wear gloves and a gown when removing the patient’s bedpan. c. Prevent the droplet spread of the organism. d. Use caution when bringing food to the patient. 27. Discharge instructions for a patient who has been operated on for colorectal cancer include irrigating the colostomy. The nurse knows her teaching is effective when the patient states he’ll contact the doctor if: a. He experiences abdominal cramping while the irrigant is infusing b. He has difficulty inserting the irrigation tube into the stoma c. He expels flatus while the return is running out d. He’s unable to complete the procedure in 1 hour 28. The nurse explains to the patient who has an abdominal perineal resection that an indwelling urinary catheter must be kept in place for several days afterward because: a. It prevents urinary tract infection following surgery b. It prevents urine retention and resulting pressure on the perineal wound c. It minimizes the risk of wound contamination by the urine d. It determines whether the surgery caused bladder trauma 29. The first day after, surgery the nurse finds no measurable fecal drainage from a patient’s colostomy stoma. What is the most appropriate nursing intervention? a. Call the doctor immediately. b. Obtain an order to irrigate the stoma. c. Place the patient on bed rest and call the doctor. d. Continue the current plan of care. 30. If a patient’s GI tract is functioning but he’s unable to take foods by mouth, the preferred method of feeding is: a. Total parenteral nutrition b. Peripheral parenteral nutrition c. Enteral nutrition d. Oral liquid supplements 31. Which type of solution causes water to shift from the cells into the plasma? a. Hypertonic b. Hypotonic c. Isotonic d. Alkaline 32. Particles move from an area of greater osmelarity to one of lesser osmolarity through: a. Active transport b. Osmosis c. Diffusion d. Filtration 33. Which assessment finding indicates dehydration?
  • 39. a. Tenting of chest skin when pinched b. Rapid filling of hand veins c. A pulse that isn’t easily obliterated d. Neck vein distention 34. Which nursing intervention would most likely lead to a hypo-osmolar state? a. Performing nasogastric tube irrigation with normal saline solution b. Weighing the patient daily c. Administering tap water enema until the return is clear d. Encouraging the patient with excessive perspiration to dink broth 35. Which assessment finding would indicate an extracellular fluid volume deficit? a. Bradycardia b. A central venous pressure of 6 mm Hg c. Pitting edema d. An orthostatic blood pressure change 36. A patient with metabolic acidosis has a preexisting problem with the kidneys. Which other organ helps regulate blood pH? a. Liver b. Pancreas c. Lungs d. heart 37. The nurse considers the patient anuric if the patient; a. Voids during the nighttime hours b. Has a urine output of less than 100 ml in 24 hours c. Has a urine output of at least 100 ml in 2 hours d. Has pain and burning on urination 38. Which nursing action is appropriate to prevent infection when obtaining a sterile urine specimen from an indwelling urinary catheter? a. Aspirate urine from the tubing port using a sterile syringe and needle b. Disconnect the catheter from the tubing and obtain urine c. Open the drainage bag and pour out some urine d. Wear sterile gloves when obtaining urine 39. After undergoing a transurethral resection of the prostate to treat benign prostatic hypertrophy, a patient is retuned to the room with continuous bladder irrigation in place. One day later, the patient reports bladder pain. What should the nurse do first? a. Increase the I.V. flow rate b. Notify the doctor immediately c. Assess the irrigation catheter for patency and drainage d. Administer meperidine (Demerol) as prescribed 40. A patient comes to the hospital complaining of sudden onset of sharp, severe pain originating in the lumbar region and radiating around the side and toward the bladder. The patient also reports nausea and vomiting and appears pale, diaphoretic, and anxious. The doctor tentatively diagnoses renal calculi and orders flat-plate abdominal X-rays. Renal calculi can form anywhere in the urinary tract. What is their most common formation site? a. Kidney b. Ureter
  • 40. c. Bladder d. Urethra 41. A patient comes to the hospital complaining of severe pain in the right flank, nausea, and vomiting. The doctor tentatively diagnoses right ureter-olithiasis (renal calculi). When planning this patient’s care, the nurse should assign highest priority to which nursing diagnosis? a. Pain b. Risk of infection c. Altered urinary elimination d. Altered nutrition: less than body requirements 42. The nurse is reviewing the report of a patient’s routine urinalysis. Which of the following values should the nurse consider abnormal? a. Specific gravity of 1.002 b. Urine pH of 3 c. Absence of protein d. Absence of glucose 43. A patient with suspected renal insufficiency is scheduled for a comprehensive diagnostic work-up. After the nurse explains the diagnostic tests, the patient asks which part of the kidney “does the work.” Which answer is correct? a. The glomerulus b. Bowman’s capsule c. The nephron d. The tubular system 44. During a shock state, the renin-angiotensin-aldosterone system exerts which of the following effects on renal function? a. Decreased urine output, increased reabsorption of sodium and water b. Decreased urine output, decreased reabsorption of sodium and water c. Increased urine output, increased reabsorption of sodium and water d. Increased urine output, decreased reabsorption of sodium and water 45. While assessing a patient who complained of lower abdominal pressure, the nurse notes a firm mass extending above the symphysis pubis. The nurse suspects: a. A urinary tract infection b. Renal calculi c. An enlarged kidney d. A distended bladder 46. Gregg Lohan, age 75, is admitted to the medical-surgical floor with weakness and left- sided chest pain. The symptoms have been present for several weeks after a viral illness. Which assessment finding is most symptomatic of pericarditis? a. Pericardial friction rub b. Bilateral crackles auscultated at the lung bases c. Pain unrelieved by a change in position d. Third heart sound (S3) 47. James King is admitted to the hospital with right-side-heart failure. When assessing him for jugular vein distention, the nurse should position him: a. Lying on his side with the head of the bed flat. b. Sitting upright.
  • 41. c. Flat on his back. d. Lying on his back with the head of the bed elevated 30 to 45 degrees. 48. The nurse is interviewing a slightly overweight 43-year-old man with mild emphysema and borderline hypertension. He admits to smoking a pack of cigarettes per day. When developing a teaching plan, which of the following should receive highest priority to help decrease respiratory complications? a. Weight reduction b. Decreasing salt intake c. Smoking cessation d. Decreasing caffeine intake 49. What is the ratio of chest compressions to ventilations when one rescuer performs cardiopulmonary resuscitation (CPR) on an adult? a. 15:1 b. 15:2 c. 12:1 d. 12:2 50. When assessing a patient for fluid and electrolyte balance, the nurse is aware that the organs most important in maintaining this balance are the: a. Pituitary gland and pancreas b. Liver and gallbladder. c. Brain stem and heart. d. Lungs and kidneys. MEDICAL SURGICAL ANSWERS: 1. Answer: D A 1+ pulse indicates weak pulses and is associated with diminished perfusion. A 4+ is bounding perfusion, a 3+ is increased perfusion, a 2+ is normal perfusion, and 0 is absent perfusion. 2. Answer: A A murmur that indicates heart disease is often accompanied by dyspnea on exertion, which is a hallmark of heart failure. Other indicators are tachycardia, syncope, and chest pain. Subcutaneous emphysema, thoracic petechiae, and perior-bital edema aren’t associated with murmurs and heart disease. 3. Answer: C Pregnancy increase plasma volume and expands the uterine vascular bed, possibly increasing both the heart rate and cardiac output. These changes may cause cardiac stress, especially during the second trimester. Blood pressure during early pregnancy may decrease, but it gradually returns to prepregnancy levels. 4. Answer: D Decreased cardiac output related to reduced myocardial contractility is the greatest threat to the survival of a patient with cardiomyopathy. The other options can be addressed once cardiac output and myocardial contractility have been restored.
  • 42. 5. Answer: D Evaluation assesses the effectiveness of the treatment plan by determining if the patient has met the expected treatment outcome. Planning refers to designing a plan of action that will help the nurse deliver quality patient care. Implementation refers to all of the nursing interventions directed toward solving the patient’s nursing problems. Analysis is the process of identifying the patient’s nursing problems. 6. Answer: B A basilar skull fracture carries the risk of complications of dural tear, causing CSF leakage and damage to cranial nerves I, II, VII, and VIII. Classic findings in this type of fracture may include otorrhea, rhinorrhea, Battle’s signs, and raccoon eyes. Surgical treatment isn’t always required. 7. Answer: A Barbiturates may be used to induce a coma in a patient with increased ICP. This decreases cortical activity and cerebral metabolism, reduces cerebral blood volume, decreases cerebral edema, and reduces the brain’s need for glucose and oxygen. Carbonic anhydrase inhibitors are used to decrease ocular pressure or to decrease the serum pH in a patient with metabolic alkalosis. Anticholinergics have many uses including reducing GI spasms. Histamine receptor blockers are used to decrease stomach acidity. 8. Answer: B Headache and projectile vomiting are early signs of increased ICP. Decreased systolic blood pressure, unconsciousness, and dilated pupils that don’t reac to light are considered late signs. 9. Answer: A Immediate bed rest is necessary to prevent further injury. Both eyes should be patched to avoid consensual eye movement and the patient should receive early referral to an ophthalmologist should treat the condition immediately. Retinal reattachment can be accomplished by surgery only. If the macula is detached or threatened, surgery is urgent; prolonged detachment of the macula results in permanent loss of central vision. 10. Answer: C A chemical burn to the eye requires immediate instillation of a topical anesthetic followed by irrigation with copious amounts of saline solution. Irrigation should be done for 5 to 10 minutes, and then the pH of the eye should be checked. Irrigation should be continued until the pH of the eye is restored to neutral (pH 7.0): Double eversion of the eyelids should be performed to look for and remove ciliary spasm, and an antibiotic ointment can be administered to reduce the risk of infection. Then the eye should be patched. Parenteral narcotic analgesia is often required for pain relief. An ophthalmologist should also be consulted. 11. Answer: B A positive response to one or both tests indicates meningeal irritation that is present with meningitis. Brudzinski’s and Kernig’s signs don’t occur in CVA, seizure disorder, or Parkinson’s disease. 12. Answer: D Gliomas account for approximately 45% of all brain tumors. Meningiomas are the second most common, with 15%. Angiomas and hemangioblastomas are types of cerebral vascular tumors that account for 3% of brain tumors.
  • 43. 13. Answer: D The patient with Parkinson’s disease may be hypersensitive to heat, which increases the risk of hyperthermia, and he should be instructed to avoid sun exposure during hot weather. 14. Answer: C Global aphasia occurs when all language functions are affected. Receptive aphasia, also known as Wernicke’s aphasia, affects the ability to comprehend written or spoken words. Expressive aphasia, also known as Broca’s aphasia, affected the patient’s ability to form language and express thoughts. Conduction aphasia refers to abnormalities in speech repetition. 15. Answer: D Patients with a history of headaches, especially migraines, should be taught to keep a food diary to identify potential food triggers. Typical headache triggers include alcohol, aged cheeses, processed meats, and chocolate and caffeine-containing products. 16. Answer: B An explosive headache or “the worst headache I’ve ever had” is typically the first presenting symptom of a bleeding cerebral aneurysm. Photophobia, seizures, and hemiparesis may occur later. 17. Answer: C An embolic injury, caused by a traveling clot, may result from atrial fibrillation. Blood may pool in the fibrillating atrium and be released to travel up the cerebral artery to the brain. Persistent hypertension may place the patient at risk for a thrombotic injury to the brain. Subarachnoid hemorrhage and skull fractures aren’t associated with emboli. 18. Answer: D Women with spinal cord injuries who were sexually active may continue having sexual intercourse and must be reminded that they can still become pregnant. She may be fully capable of achieving orgasm. An indwelling urinary catheter may be left in place during sexual intercourse. Positioning will need to be adjusted to fit the patient’s needs. 19. Answer: A The nurse must have a wrench taped on the vest at all times for quick halo removal in emergent situations. The brace isn’t to be removed for any other reason until the cervical fracture is healed. Placing a pillow under the patient’s head may alter the stability of the brace. 20. Answer: D Nimodipine is a calcium channel blocker that acts on cerebral blood vessels to reduce vasospasm. The drug doesn’t increase the amount of calcium, affect cerebral vasculature growth, or reduce cerebral oxygen demand. 21. Answer: D Men with spinal cord injury should be taught that the higher the level of the lesion, the better their sexual function will be. The sacral region is the lowest area on the spinal column and injury to this area will cause more erectile dysfunction. 22. Answer: C Tegretol should be taken with food to minimize GI distress. Taking it at meals will also establish a regular routine, which should help compliance.
  • 44. 23. Answer: B Patient with pneumococcal meningitis require respiratory isolation for the first 24 hours after treatment is initiated. 24. Answer: A Early symptoms of ALS include fatigue while talking, dysphagia, and weakness of the hands and arms. ALS doesn’t cause a change in mental status, paresthesia, or fractures. 25. Answer: C Increased pressure within the portal veins causes them to bulge, leading to rupture and bleeding into the lower esophagus. Bleeding associated with esophageal varices doesn’t stem from esophageal perforation, pulmonary hypertension, or peptic ulcers. 26. Answer: B The nurse should wear gloves and a gown when removing the patient’s bedpan because the type A hepatitis virus occurs in stools. It may also occur in blood, nasotracheal secretions, and urine. Type A hepatitis isn’t transmitted through the air by way of droplets. Special precautions aren’t needed when feeding the patient, but disposable utensils should be used. 27. Answer: B The patient should notify the doctor if he has difficulty inserting the irrigation tube into the stoma. Difficulty with insertion may indicate stenosis of the bowel. Abdominal cramping and expulsion of flatus may normally occur with irrigation. The procedure will often take an hour to complete. 28. Answer: B An indwelling urinary catheter is kept in place several days after this surgery to prevent urine retention that could place pressure on the perineal wound. An indwelling urinary catheter may be a source of postoperative urinary tract infection. Urine won’t contaminate the wound. An indwelling urinary catheter won’t necessarily show bladder trauma. 29. Answer: D The colostomy may not function for 2 days or more (48 to 72 hours) after surgery. Therefore, the normal plan of care can be followed. Since no fecal drainage is expected for 48 to 72 hours after a colostomy (only mucous and serosanguineous), the doctor doesn’t have to be notified and the stoma shouldn’t be irrigated at this time. 30. Answer: C If the patient’s GI tract is functioning, enteral nutrition via a feeding tube is the preferred method. Peripheral and total parenteral nutrition places the patient at risk for infection. If the patient is unable to consume foods by mouth, oral liquid supplements are contraindicated. 31. Answer: A A hypertonic solution causes water to shift from the cells into the plasma because the hypertonic solution has a greater osmotic pressure than the cells. A hypotonic solution has a lower osmotic pressure than that of the cells. It causes fluid to shift into the cells, possibly resulting in rupture. An isotonic solution, which has the same osmotic pressure as the cells, wouldn’t cause any shift. A solution’s alkalinity is related to the hydrogen ion concentration, not its osmotic effect. 32. Answer: C Particles move from an area of greater osmolarity to one of lesser osmolarity through
  • 45. diffusion. Active transport is the movement of particles though energy expenditure from other sources such as enzymes. Osmosis is the movement of a pure solvent through a semipermeable membrane from an area of greater osmolarity to one of lesser osmolarity until equalization occurs. The membrane is impermeable to the solute but permeable to the solvent. Filtration is the process by which fluid is forced through a membrane by a difference in pressure; small molecules pass through, but large ones don’t. 33. Answer: A Tenting of chest skin when pinched indicates decreased skin elasticity due to dehydration. Hand veins fill slowly with dehydration, not rapidly. A pulse that isn’t easily obliterated and neck vein distention indicate fluid overload, not dehydration. 34. Answer: C Administering a tap water enema until return is clear would most likely contribute to a hypo-osmolar state. Because tap water is hypotonic, it would be absorbed by the body, diluting the body fluid concentration and lowering osmolarity. Weighing the patient is the easiest, most accurate method to determine fluid changes. Therefore, it helps identify rather than contribute to fluid imbalance. Nasogastric tube irrigation with normal saline solution wouldn’t cause a shift in fluid balance. Drinking broth wouldn’t contribute to a hypo-osmolar state because it doesn’t replace sodium and water lost through excessive perspiration. 35. Answer: D An orthostatic blood pressure indicates an extracellular fluid volume deficit. (The extracellular compartment consists of both the intravascular compartment and interstitial space.) A fluid volume deficit within the intravascular compartment would cause tachycardia, not bradycardia or orthostatic blood pressure change. A central venous pressure of 6 mm Hg is in the high normal range, indicating adequate hydration. Pitting edema indicates fluid volume overload. 36. Answer: C The respiratory and renal systems act as compensatory mechanisms to counteract-base imbalances. The lungs alter the carbon dioxide levels in the blood by increasing or decreasing the rate and depth of respirations, thereby increasing or decreasing carbon dioxide elimination. The liver, pancreas, and heart play no part in compensating for acid- base imbalances. 37. Answer: B Anuria refers to a urine output of less than 100 ml in 24 hours. The baseline for urine output and renal function is 30 ml of urine per hour. A urine output of at least 100 ml in 2 hours is within normal limits. Voiding at night is called nocturia. Pain and burning on urination is called dysuria. 38. Answer: A To obtain urine properly, the nurse should aspirate it from a port, using a sterile syringe after cleaning the port. Opening a closed urine drainage system increases the risk of urinary tract infection. Standard precautions specify the use of gloves during contract with body fluids; however, sterile gloves aren’t necessary. 39. Answer: C Although postoperative pain is expected, the nurse should ensure that other factors, such as an obstructed irrigation catheter, aren’t the cause of the pain. After assessing catheter patency, the nurse should administer an analgesic such as meperidine as prescribed.
  • 46. Increasing the I.V. flow rate may worse the pain. Notifying the doctor isn’t necessary unless the pain is severe or unrelieved by the prescribed medication. 40. Answer: A Renal calculi most commonly from in the kidney. They may remain there or become lodged anywhere along the urinary tract. The ureter, bladder, and urethra are less common sites of renal calculi formation. 41. Answer: A Ureterolithiasis typically causes such acute, severe pain that the patient can’t rest and becomes increasingly anxious. Therefore, the nursing diagnosis of pain takes highest priority. Risk for infection and altered urinary elimination are appropriate once the patient’s pain is controlled. Altered nutrition: less than body requirements isn’t appropriate at this time. 42. Answer: B Normal urine pH is 4.5 to 8; therefore, a urine pH of 3 is abnormal and may indicate such conditions as renal tuberculosis, pyrexia, phenylketonuria, alkaptonuria, and acidosis. Urine specific gravity normally ranges from 1.002 to 1.032, making the patient’s value normal. Normally, urine contains no protein, glucose, ketones, bilirubin, bacteria, casts, or crystals. 43. Answer: C The nephron is the kidney’s functioning unit. The glomerulus, Bowman’s capsule, and tubular system are components of the nephron. 44. Answer: A As a response to shock, the renin-angiotensin-aldosterone system alters renal function by decreasing urine output and increasing reabsorption of sodium and water. Reduced renal perfusion stimulates the renin-angiotensin-aldosterone system in an effort to conserve circulating volume. 45. Answer: D The bladder isn’t usually palpable unless it is distended. The feeling of pressure is usually relieved with urination. Reduced bladder tone due to general anesthesia is a common postoperative complication that causes difficulty in voiding. A urinary tract infection and renal calculi aren’t palpable. The kidneys aren’t palpable above the symphysis pubis. 46. Answer: A A pericardial friction rub may be present with the pericardial effusion of pericarditis. The lungs are typically clear when auscultated. Sitting up and leaning forward often relieves pericarditis pain. An S3 indicates left-sided heart failure and isn’t usually present with pericarditis. 47. Answer: D Assessing jugular vein distention should be done when the patient is in semi-Fowler’s position (head of the bed elevated 30 to 45 degrees). If the patient lies flat, the veins will be more distended; if he sits upright, the veins will be flat. 48. Answer: C Smoking should receive highest priority when trying to reduce risk factors for with respiratory complications. Losing weight and decreasing salt and caffeine intake can help to decrease risk factors for hypertension. 49. Answer: B The correct ratio of compressions to ventilations when one rescuer performs CPR is 15:2
  • 47. 50. Answer: D The lungs and kidneys are the body’s regulators of homeostasis. The lungs are responsible for removing fluid and carbon dioxide; the kidneys maintain a balance of fluid and electrolytes. The other organs play secondary roles in maintaining homeostasis. PSYCHIATRIC NSG 1. Which of the following medications would the nurse in-charge expect the doctor to order to reverse a dystonic reaction? a. Procholorperazine (Compazine) b. Diphenhydramine (Benadryl) c. Haloperidol (Haldol) d. Midazolam (Versed) 2. While pacing in the hall, a female patient with paranoid schizophrenia runs to the nurse and says, “Why are you poisoning me? I know you work for central thought control! You can keep my thoughts. Give me back my soul!” how should the nurse respond? a. “I’m a nurse, I’m not poisoning you. It’s against the nursing code of ethics.” b. “I’m a nurse, and you’re a patient in the hospital. I’m not going to harm you.” c. “I’m not poisoning you. And how could I possibly steal your soul?” d. “I sense anger, Are you feeling angry today?” 3. After completing chemical detoxification and a 12-step program to treat crack addiction, a male patient is being prepared for discharge. Which remark by the patient indicates a realistic view of the future? a. “I’m never going to use crack again.” b. “I know what I have to do. I have to limit my crack use.” c. “I’m going to take 1 day at a time. I’m not making any promises.” d. “I can’t touch crack again, but I sure could use a drink. I’ve earned it.”
  • 48. 4. The nurse formulates a nursing diagnosis of “impaired verbal communication” fora male patient with schizotypal personality disorder. Based on this nursing diagnosis, which nursing intervention is most appropriate? a. Helping the patient to participate in social interactions b. Establishing a one-on-one relationship with the patient c. Establishing alternative forms of communication d. Allowing the patient to decide when he wants to participate in verbal communication with you 5. A female patient with obsessive-compulsive disorder tells the nurse that he must check the lock on his apartment door 25 times before leaving for an appointment. The nurse knows that this behavior represents the patient’s attempt to: a. Call attention to himself b. Control his thoughts c. Maintain the safety of his home d. Reduce anxiety 6. A patient, age 42, is admitted for surgical biopsy of a suspicious lump in her left breast. When the nurse comes to her surgery, she is tearfully finishing a letter to her children. She tells the nurse, “I want to leave this for my children in case anything goes wrong today. “Which response by the nurse would be most therapeutic? a. “In case anything goes wrong? What are your thoughts and feelings right now?” b. “I can’t understand that you’re nervous, but this is really a minor procedure. You’ll be back in your room before you know it.” c. “Try to take a few deep breaths and relax. I have some medication that will help.” d. “I’m sure your children know how much you love them. You’ll be able to talk to them on the phone in a few hours.” 7. Which nursing intervention is most important when restraining a violent male patient? a. Reviewing hospital policy regarding how long the patient can be restrained b. Preparing a p.r.n. dose of the patient’s psychotropic medication
  • 49. c. Checking that the restraints have been applied correctly d. Asking if the patient needs to use the bathroom or is thirsty 8. How soon after chlorpromazine administration should the nurse in charge expect to see a patient’s delusion thoughts and hallucinations eliminated? a. Several minutes b. Several hours c. Several days d. Several weeks 9. Mental health laws in each state specify when restraints can be used and which type of restraints are allowed. Most laws stipulate that restraints can be used: a. For a maximum of 2 hours b. As necessary to control the patient c. If the patient poses a present danger to self or others d. Only with the patient’s consent 10. A female patient has been severely depressed since her husband died 6 months ago. Her doctor prescribes amitriptyline hydrochloride (Elavil), 50 mg P.O. daily. Before administering amitriptyline, the nurse reviews the patient’s medical history. Which preexisting condition would require cautions use of this drug? a. Hiatal hernia b. Hypernatremia c. Hepatic disease d. Hypokalemia 11. The physician orders a new medication for a male client with generalized anxiety disorder. During medication teaching, which statement or question by the nurse would be most appropriate? a. “Take this medication. It will reduce your anxiety.” b. “Do you have any concern about taking the medication?”